Найди верное неравенство: Верные и неверные равенства и неравенства — урок. Математика, 1 класс.

Содержание

Тест по математике для 4 класса

                     Вар-т 1            Вар-т 2            Вар-т 3

1. Найдите продолжение ряда чисел 98700, 87600, 76500, ….

а). 65400, 54300
б). 65400, 76500
в). 5400, 54300

2. Найди числа, которые в числовом ряду расположены между числами 397 и 401.

а). 397, 398, 399;
б). 398, 399, 400;
в). 399, 400, 401.

3. Разложи на разрядные слагаемые число 5731.

а). 5731 = 5000 + 730 + 1
б). 5731 = 5000 + 700 + 30 + 1
в). 5731 = 5000 + 700 + 31

4.Какие цифры нужно вставить на место пропусков, чтобы неравенство 170308<170..8 было истинным.

а). 31
б). 29
в). 30

5. Найди число, которое на 1 меньше числа 80000.

а). 70000
б). 79999
в). 7991

6. Сумма трёх слагаемых равна 623. Первое слагаемое – 20, второе слагаемое – 3. Найдите третье слагаемое.

а). 600
б). 60
в). 6000

7. Найдите верное неравенство.

а). 7130>7310
б). 371<317
в). 1370<1730

8. Найдите верную запись.

а). 38574=38 тыс. 574 ед.
б). 38574=385 тыс. 74 ед.
в). 38574=3 тыс. 8574 ед.

9. Найди число, которое на 1 десяток больше числа 80020.

а). 80030
б). 80120
в). 80021

10. Найдите число, в котором число десятков в 2 раза больше, чем число сотен.

а). 841
б). 481
в). 184

11. Сумма трёх слагаемых равна 6623. Первое слагаемое – 6000, второе слагаемое – 600.
Найдите третье слагаемое.

12. Найдите число, в котором число сотен меньше числа десятков в 2 раза, а число десятков на 3 больше числа единиц.

13. Найди число, которое меньше числа 88223 на 1 сотню и 1 единицу.

14. Произведение трёх множителей — 60000.

Первый множитель – 100, второй – 20.
Найдите третий множитель.

15. Вставь пропущенное число, чтобы неравенство было верным: … : 10 < 3705

16. На сколько надо уменьшить 1200, чтобы получить значение произведения 160 и 4

17. Запиши число, в котором число десятков тысяч больше числа единиц тысяч

18. Из 100 кг свеклы при переработке получают 16 кг сахара.
Сколько сахара можно получить из 1 тонны свеклы?

19. Сумма двух чисел равна 462.
Одно из них оканчивается нулём.
Если этот нуль зачеркнуть, то получится второе число.
Найди эти числа.

20. Сумма трёх чисел 30212.
Первое слагаемое — наименьшее пятизначное число, второе – наибольшее четырёхзначное число.

Найди разность третьего слагаемого и числа 7539

Тест по математике для 4 класса


                     Вар-т 1            Вар-т 2            Вар-т 3

тест | Тест по математике (3 класс) по теме:

1 вариант

1.Найдите число, в котором 7 единиц II разряда:

А) 709                    б) 607                   в)576

2.Какое число при счёте следует за числом 679?

А) 669                             б) 579                             в) 680

3.Какое число состоит из 8 сотен и 6 единиц?

А) 860                      б) 680                             в)806

4.Найди число, которое можно записать в виде суммы разрядных слагаемых та: 600 + 80 + 4.

А) 486                                б) 684                                        в) 864

5.Найди верное неравенство:

А) 613 < 316                                                   б) 631 < 613                         в) 163 > 136

6.Найди число, которое меньше 5 сотен на 1:

А) 400                    б) 499                            в) 490

7.Сколько надо прибавить к числу 400, чтобы получить 480?

А) 80                      б) 8                          в) 800

8.Сумма трёх слагаемых 934. Первое слагаемое 30, второе – 4. Найди третье слагаемое.

А) 9                     б) 90                                       в)900

9.Сравни числа: 280 … 208

А) >                              б) <                                     в) =

10.Число 560 уменьши на частное чисел 420 и 7.

А) 20                                б) 500                                 в) 140

2 вариант

1.Найдите число, в котором 8 единиц II разряда:

А) 807                    б) 709                   в)780

2.Какое число при счёте следует за числом 489?

А) 479                             б) 389                             в) 490

3.Какое число состоит из 9 сотен и 7 единиц?

А) 790                      б) 709                             в)907

4.Найди число, которое можно записать в виде суммы разрядных слагаемых та: 800 + 30 + 5.

А) 538                                б) 853                                        в) 835

5.Найди верное неравенство:

А) 815 < 851                                                   б) 581 < 518                         в) 185 > 158

6.Найди число, которое меньше 7 сотен на 1:

А) 600                    б) 699                            в) 690

7.Сколько надо прибавить к числу 800, чтобы получить 870?

А) 7                      б) 70                          в) 700

8.Сумма трёх слагаемых 736. Первое слагаемое 30, второе – 6. Найди третье слагаемое.

А) 7                     б) 70                                       в)700

9.Сравни числа: 137 … 170

А) >                              б) <                                     в) =

10.Из произведения чисел 60 и 5 вычти число 154.

А) 164                                б) 146                                 в) 150

Общие сведения о неравенствах

Данный материал может показаться сложным для понимания. Рекомендуется изучать его маленькими частями.

Предварительные навыки

Определения и свойства

Неравенством мы будем называть два числовых или буквенных выражения, соединенных знаками >, <, ≥, ≤ или ≠.

Пример: 5 > 3

Данное неравенство говорит о том, что число 5 больше, чем число 3. Острый угол знака неравенства должен быть направлен в сторону меньшего числа. Это неравенство является верным, поскольку 5 больше, чем 3.

Если на левую чашу весов положить арбуз массой 5 кг, а на правую — арбуз массой 3 кг, то левая чаша перевесит правую, и экран весов покажет, что левая чаша тяжелее правой:

Если 5 > 3, то 3 < 5. То есть левую и правую часть неравенства можно поменять местами, изменив знак неравенства на противоположный. В ситуации с весами: большой арбуз можно положить на правую чашу, а маленький арбуз на левую. Тогда правая чаша перевесит левую, и экран покажет знак <

Если в неравенстве 5 > 3, не трогая левую и правую часть, поменять знак на <, то получится неравенство 5 < 3. Это неравенство не является верным, поскольку число 3 не может быть больше числа 5.

Числа, которые располагаются в левой и правой части неравенства, будем называть членами этого неравенства. Например, в неравенстве 5 > 3 членами являются числа 5 и 3.

Рассмотрим некоторые важные свойства для неравенства 5 > 3.
В будущем эти свойства будут работать и для других неравенств.

Свойство 1.

Если к левой и правой части неравенства 5 > 3 прибавить или вычесть одно и то же число, то знак неравенства не изменится.

Например, прибавим к обеим частям неравенства число 4. Тогда получим:

Видим, что левая часть по-прежнему больше правой.

Теперь попробуем вычесть из обеих частей неравенства 5 > 3 какое-нибудь число, скажем число 2

Видим, что левая часть по-прежнему больше правой.

Из данного свойства следует, что любой член неравенства можно перенести из одной части в другую часть, изменив знак этого члена. Знак неравенства при этом не изменится.

Например, перенесём в неравенстве 5 > 3, член 5 из левой части в правую часть, изменив знак этого члена. После переноса члена 5 в правую часть, в левой части ничего не останется, поэтому запишем там 0

0 > 3 − 5

0 > −2

Видим, что левая часть по-прежнему больше правой.


Свойство 2.

Если обе части неравенства умножить или разделить на одно и то же положительное число, то знак неравенства не изменится.

Например, умножим обе части неравенства 5 > 3 на какое-нибудь положительное число, скажем на число 2. Тогда получим:

Видим, что левая часть по-прежнему больше правой.

Теперь попробуем разделить обе части неравенства 5 > 3 на какое-нибудь число. Разделим их на 2

Видим, что левая часть по-прежнему больше правой.

Свойство 3.

Если обе части неравенства умножить или разделить на одно и то же отрицательное число, то знак неравенства изменится на противоположный.

Например, умножим обе части неравенства 5 > 3 на какое-нибудь отрицательное число, скажем на число −2. Тогда получим:

Видим, что левая часть стала меньше правой. То есть знак неравенства изменился на противоположный.

Теперь попробуем разделить обе части неравенства 5 > 3 на какое-нибудь отрицательное число. Давайте разделим их на −1

Видим, что левая часть стала меньше правой. То есть знак неравенства изменился на противоположный.

Само по себе неравенство можно понимать, как некоторое условие. Если условие выполняется, то неравенство является верным. И наоборот, если условие не выполняется, то неравенство не верно.

Например, чтобы ответить на вопрос является ли верным неравенство 7 > 3, нужно проверить выполняется ли условие

«больше ли 7, чем 3». Мы знаем, что число 7 больше, чем число 3. То есть условие выполнено, а значит и неравенство 7 > 3 верно.

Неравенство 8 < 6 не является верным, поскольку не выполняется условие «8 меньше, чем 6».

Другим способом определения верности неравенства является составление разности из левой и правой части данного неравенства. Если разность положительна, то левая часть больше правой части. И наоборот, если разность отрицательна, то левая часть меньше правой части. Более точно это правило выглядит следующим образом:

Число a больше числа b, если разность a − b положительна. Число a меньше числа b

, если разность a − b отрицательна.

Например, мы выяснили, что неравенство 7 > 3 является верным, поскольку число 7 больше, чем число 3. Докажем это с помощью правила, приведённого выше.

Составим разность из членов 7 и 3. Тогда получим 7 − 3 = 4. Согласно правилу, число 7 будет больше числа 3, если разность 7 − 3 окажется положительной. У нас она равна 4, то есть разность положительна. А значит число 7 больше числа 3.

Проверим с помощью разности верно ли неравенство 3 < 4. Составим разность, получим 3 − 4 = −1. Согласно правилу, число 3 будет меньше числа 4, если разность 3 − 4 окажется отрицательной. У нас она равна −1, то есть разность отрицательна. А значит число 3 меньше числа 4.

Проверим верно ли неравенство 5 > 8. Составим разность, получим 5 − 8 = −3. Согласно правилу, число 5 будет больше числа 8, если разность 5 − 8 окажется положительной. У нас разность равна −3, то есть она не является положительной. А значит число 5 не больше числа 8. Иными словами, неравенство 5 > 8 не является верным.


Строгие и нестрогие неравенства

Неравенства, содержащие знаки >, < называют строгими. А неравенства, содержащие знаки ≥, ≤  называют нестрогими.

Примеры строгих неравенства мы рассматривали ранее. Таковыми являются неравенства 5 > 3, 7 < 9.

Нестрогим, например, является неравенство 2 ≤ 5. Данное неравенство читают следующим образом:

«2 меньше или равно 5».

Запись 2 ≤ 5 является неполной. Полная запись этого неравенства выглядит следующим образом:

2 < 5 или 2 = 5

Тогда становится очевидным, что неравенство 2 ≤ 5 состоит из двух условий: «два меньше пять» и «два равно пять».

Нестрогое неравенство верно в том случае, если выполняется хотя бы одно из его условий. В нашем примере верным является условие «2 меньше 5». Значит и само неравенство 2 ≤ 5 верно.

Пример 2. Неравенство 2 ≤ 2 является верным, поскольку выполняется одно из его условий, а именно 2 = 2.

Пример 3. Неравенство 5 ≤ 2 не является верным, поскольку не выполняется ни одно из его условий: ни 5 < 2 ни 5 = 2.


Двойное неравенство

Число 3 больше, чем число 2 и меньше, чем число 4. В виде неравенства это высказывание можно записать так: 2 < 3 < 4. Такое неравенство называют двойным.

Двойное неравенство может содержать знаки нестрогих неравенств. К примеру, если число 5 больше или равно, чем число 2, и меньше или равно, чем число 7, то можно записать, что 2 ≤ 5 ≤ 7

Чтобы правильно записать двойное неравенство, сначала записывают член находящийся в середине, затем член находящийся слева, затем член находящийся справа.

Например, запишем, что число 6 больше, чем число 4, и меньше, чем число 9.

Сначала записываем 6

Слева записываем, что это число больше, чем число 4

Справа записываем, что число 6 меньше, чем число 9


Неравенство с переменной

Неравенство, как и равенство может содержать переменную.

Например, неравенство x > 2 содержит переменную x. Обычно такое неравенство нужно решить, то есть выяснить при каких значениях x данное неравенство становится верным.

Решить неравенство означает найти такие значения переменной x, при которых данное неравенство становится верным.

Значение переменной, при котором неравенство становится верным, называется решением неравенства.

Неравенство > 2 становится верным при x = 3, x = 4, x = 5, x = 6 и так далее до бесконечности. Видим, что это неравенство имеет не одно решение, а множество решений.

Другими словами, решением неравенства x > 2 является множество всех чисел, бóльших 2. При этих числах неравенство будет верным. Примеры:

3 > 2

4 > 2

5 > 2

Число 2, располагающееся в правой части неравенства x > 2, будем называть границей данного неравенства. В зависимости от знака неравенства, граница может принадлежать множеству решений неравенства либо не принадлежать ему.

В нашем примере граница неравенства не принадлежит множеству решений, поскольку при подстановке числа 2 в неравенство x > 2 получается не верное неравенство 2 > 2. Число 2 не может быть больше самого себя, поскольку оно равно самому себе (2 = 2).

Неравенство x > 2 является строгим. Его можно прочитать так: «x строго больше 2″. То есть все значения, принимаемые переменной x должны быть строго больше 2. В противном случае, неравенство верным не будет.

Если бы нам было дано нестрогое неравенство ≥ 2, то решениями данного неравенства были бы все числа, которые больше 2, в том числе и само число 2. В этом неравенстве граница 2 принадлежит множеству решений неравенства, поскольку при подстановке числа 2 в неравенство x ≥ 2 получается верное неравенство 2 ≥ 2. Ранее было сказано, что нестрогое неравенство является верным, если выполняется хотя бы одно из его условий. В неравенстве 2 ≥ 2 выполняется условие 2 = 2, поэтому и само неравенство 2 ≥ 2 верно.


Как решать неравенства

Процесс решения неравенств во многом схож с процессом решения уравнений. При решении неравенств мы будем применять свойства, которые изучили вначале данного урока, такие как: перенос слагаемых из одной части неравенства в другую часть, меняя знак; умножение (или деление) обеих частей неравенства на одно и то же число.

Эти свойства позволяют получить неравенство, которое равносильно исходному. Равносильными называют неравенства, решения которых совпадают.

Решая уравнения мы выполняли тождественные преобразования до тех пор, пока в левой части уравнения не оставалась переменная, а в правой части значение этой переменной (например: x = 2, x = 5). Иными словами, заменяли исходное уравнение на равносильное ему уравнение до тех пор, пока не получалось уравнение вида x = a, где a значение переменной x. В зависимости от уравнения, корней могло быть один, два, бесконечное множество, либо не быть совсем.

А при решении неравенств мы будем заменять исходное неравенство на равносильное ему неравенство до тех пор, пока в левой части не останется переменная этого неравенства, а в правой части его граница.

Пример 1. Решить неравенство 2> 6

Итак, нужно найти такие значения x, при подстановке которых в 2> 6 получится верное неравенство.

Вначале данного урока было сказано, что если обе части неравенства разделить на какое-нибудь положительное число, то знак неравенства не изменится. Если применить это свойство к неравенству, содержащему переменную, то получится неравенство равносильное исходному.

В нашем случае, если мы разделим обе части неравенства 2> 6 на какое-нибудь положительное число, то получится неравенство, которое равносильно исходному неравенству 2> 6. 

Итак, разделим обе части неравенства на 2.

В левой части осталась переменная x, а правая часть стала равна 3. Получилось равносильное неравенство > 3. На этом решение завершается, поскольку в левой части осталась переменная, а в правой части граница неравенства.

Теперь можно сделать вывод, что решениями неравенства > 3 являются все числа, которые больше 3. Это числа 4, 5, 6, 7 и так далее до бесконечности. При этих значениях неравенство > 3 будет верным.

4 > 3

5 > 3

6 > 3

7 > 3

Отметим, что неравенство > 3 является строгим. «Переменная x строго больше трёх».

А поскольку неравенство > 3 равносильно исходному неравенству 2> 6, то их решения будут совпадать. Иначе говоря, значения, которые подходят неравенству > 3, будут подходить и неравенству 2> 6. Покажем это.

Возьмём, например, число 5 и подставим его сначала в полученное нами равносильное неравенство > 3, а потом в исходное 2> 6.

Видим, что в обоих случаях получается верное неравенство.

После того, как неравенство решено, ответ нужно записать в виде так называемого числового промежутка следующим образом:

В этом выражении говорится, что значения, принимаемые переменной x, принадлежат числовому промежутку от трёх до плюс бесконечности.

Иначе говоря, все числа, начиная от трёх до плюс бесконечности являются решениями неравенства > 3. Знак  в математике означает бесконечность.

Учитывая, что понятие числового промежутка очень важно, остановимся на нём подробнее.


Числовые промежутки

Числовым промежутком называют множество чисел на координатной прямой, которое может быть описано с помощью неравенства.

Допустим, мы хотим изобразить на координатной прямой множество чисел от 2 до 8. Для этого сначала на координатной прямой отмечаем точки с координатами 2 и 8, а затем выделяем штрихами ту область, которая располагается между координатами 2 и 8. Эти штрихи будут играть роль чисел, располагающихся между числами 2 и 8

Числа 2 и 8 назовём границами числового промежутка. Рисуя числовой промежуток, точки для его границ изображают не в виде точек как таковых, а в виде кружков, которые можно разглядеть.

Границы могут принадлежать числовому промежутку либо не принадлежать ему.

Если границы не принадлежат числовому промежутку, то они изображаются на координатной прямой в виде пустых кружков.

Если границы принадлежат числовому промежутку, то кружки необходимо закрасить.

На нашем рисунке кружки были оставлены пустыми. Это означало, что границы 2 и 8 не принадлежат числовому промежутку. Значит в наш числовой промежуток будут входить все числа от 2 до 8, кроме чисел 2 и 8.

Если мы хотим включить границы 2 и 8 в числовой промежуток, то кружки необходимо закрасить:

В данном случае в числовой промежуток будут входить все числа от 2 до 8, включая числа 2 и 8.

На письме числовой промежуток обозначается указанием его границ с помощью круглых или квадратных скобок.

Если границы не принадлежат числовому промежутку, то границы обрамляются круглыми скобками.

Если границы принадлежат числовому промежутку, то границы обрамляются квадратными скобками.

На рисунке представлено два числовых промежутка от 2 до 8 с соответствующими обозначениями:

На первом рисунке числовой промежуток обозначен с помощью круглых скобок, поскольку границы 2 и 8 не принадлежат этому числовому промежутку.

На втором рисунке числовой промежуток обозначен с помощью квадратных скобок, поскольку границы 2 и 8 принадлежат этому числовому промежутку.

С помощью числовых промежутков можно записывать ответы к неравенствам. Например, ответ к двойному неравенству 2 ≤ ≤ 8 записывается так:

x ∈ [ 2 ; 8 ]

То есть сначала записывают переменную, входящую в неравенство, затем с помощью знака принадлежности ∈ указывают к какому числовому промежутку принадлежат значения этой переменной. В данном случае выражение x ∈ [ 2 ; 8 ] указывает на то, что переменная x, входящая в неравенство 2 ≤ ≤ 8, принимает все значения в промежутке от 2 до 8 включительно. При этих значениях неравенство будет верным.

Обратим внимание на то, что ответ записан с помощью квадратных скобок, поскольку границы неравенства 2 ≤ ≤ 8, а именно числа 2 и 8 принадлежат множеству решений этого неравенства.

Множество решений неравенства 2 ≤ ≤ 8 также можно изобразить с помощью координатной прямой:

Здесь границы числового промежутка 2 и 8 соответствуют границам неравенства 2 ≤ ≤ 8, а выделенная штрихами область соответствует множеству значений x, которые являются решениями неравенства 2 ≤ ≤ 8.

В некоторых источниках границы, которые не принадлежат числовому промежутку, называют открытыми.

Открытыми их называют по той причине, что числовой промежуток остаётся открытым из-за того, что его границы не принадлежат этому числовому промежутку. Пустой кружок на координатной прямой математики называют выколотой точкой. Выколоть точку значит исключить её из числового промежутка или из множества решений неравенства.

А в случае, когда границы принадлежат числовому промежутку, их называют закрытыми (или замкнутыми), поскольку такие границы закрывают (замыкают) собой числовой промежуток. Закрашенный кружок на координатной прямой также говорит о закрытости границ.

Существуют разновидности числовых промежутков. Рассмотрим каждый из них.

Числовой луч

Числовым лучом называют числовой промежуток, который задаётся неравенством x ≥ a, где a — граница данного неравенства, x — решение неравенства.

Пусть = 3. Тогда неравенство x ≥ a примет вид ≥ 3. Решениями данного неравенства являются все числа, которые больше 3, включая само число 3.

Изобразим числовой луч, заданный неравенством ≥ 3, на координатной прямой. Для этого отметим на ней точку с координатой 3, а всю оставшуюся справа от неё область выделим штрихами. Выделяется именно правая часть, поскольку решениями неравенства ≥ 3 являются числа, бóльшие 3. А бóльшие числа на координатной прямой располагаются правее

Здесь точка 3 соответствует границе неравенства ≥ 3, а выделенная штрихами область соответствует множеству значений x, которые являются решениями неравенства ≥ 3.

Точка 3, являющаяся границей числового луча, изображена в виде закрашенного кружка, поскольку граница неравенства ≥ 3 принадлежит множеству его решений.

На письме числовой луч, заданный неравенством x ≥ a, обозначается следующим образом:

[ ; +∞ )

Видно, что с одной стороны граница обрамлена квадратной скобкой, а с другой круглой. Это связано с тем, что одна граница числового луча принадлежит ему, а другая нет, поскольку бесконечность сама по себе границ не имеет и подразумевается, что по ту сторону нет числа, замыкающего этот числовой луч.

Учитывая то, что одна из границ числового луча закрыта, данный промежуток часто называют закрытым числовым лучом.

Запишем ответ к неравенству ≥ 3 с помощью обозначения числового луча. У нас переменная a равна 3

x ∈  [ 3 ; +∞ )

В этом выражении говорится, что переменная x, входящая в неравенство ≥ 3, принимает все значения от 3 до плюс бесконечности.

Иначе говоря, все числа от 3 до плюс бесконечности, являются решениями неравенства ≥ 3. Граница 3 принадлежит множеству решений, поскольку неравенство ≥ 3 является нестрогим.

Закрытым числовым лучом также называют числовой промежуток, который задаётся неравенством x ≤ a. Решениями неравенства x ≤ a являются все числа, которые меньше a, включая само число a. 

К примеру, если = 2, то неравенство примет вид ≤ 2. На координатной прямой граница 2 будет изображаться закрашенным кружком, а вся область, находящаяся слева, будет выделена штрихами. В этот раз выделяется левая часть, поскольку решениями неравенства ≤ 2 являются числа, меньшие 2. А меньшие числа на координатной прямой располагаются левее

Здесь точка 2 соответствует границе неравенства ≤ 2, а выделенная штрихами область соответствует множеству значений x, которые являются решениями неравенства ≤ 2.

Точка 2, являющаяся границей числового луча, изображена в виде закрашенного кружка, поскольку граница неравенства ≤ 2 принадлежит множеству его решений.

Запишем ответ к неравенству ≤ 2 с помощью обозначения числового луча:

x ∈  ( −∞ ; 2 ]

В этом выражении говорится, что все числа от минус бесконечности до 2, являются решениями неравенства ≤ 2. Граница 2 принадлежит множеству решений, поскольку неравенство ≤ 2 является нестрогим.

Открытый числовой луч

Открытым числовым лучом называют числовой промежуток, который задаётся неравенством x > a, где a — граница данного неравенства, x — решение неравенства.

Открытый числовой луч во многом похож на закрытый числовой луч. Различие в том, что граница a не принадлежит промежутку, как и граница неравенства x > a не принадлежит множеству его решений.

Пусть = 3. Тогда неравенство примет вид > 3. Решениями данного неравенства являются все числа, которые больше 3, за исключением числа 3

На координатной прямой граница открытого числового луча, заданного неравенством > 3, будет изображаться в виде пустого кружка. Вся область, находящаяся справа, будет выделена штрихами:

Здесь точка 3 соответствует границе неравенства x > 3, а выделенная штрихами область соответствует множеству значений x, которые являются решениями неравенства x > 3. Точка 3, являющаяся границей открытого числового луча, изображена в виде пустого кружка, поскольку граница неравенства x > 3 не принадлежит множеству его решений.

На письме открытый числовой луч, заданный неравенством x > a, обозначается следующим образом:

( ; +∞ )

Круглые скобки указывают на то, что границы открытого числового луча не принадлежат ему.

Запишем ответ к неравенству x > 3 с помощью обозначения открытого числового луча:

x ∈  ( 3 ; +∞ )

В этом выражении говорится, что все числа от 3 до плюс бесконечности, являются решениями неравенства x > 3. Граница 3 не принадлежит множеству решений, поскольку неравенство x > 3 является строгим.

Открытым числовым лучом также называют числовой промежуток, который задаётся неравенством x < a, где a — граница данного неравенства, x — решение неравенства. Решениями неравенства x < a являются все числа, которые меньше a, исключая число a. 

К примеру, если = 2, то неравенство примет вид x < 2. На координатной прямой граница 2 будет изображаться пустым кружком, а вся область, находящаяся слева, будет выделена штрихами:

Здесь точка 2 соответствует границе неравенства x < 2, а выделенная штрихами область соответствует множеству значений x, которые являются решениями неравенства x < 2. Точка 2, являющаяся границей открытого числового луча, изображена в виде пустого кружка, поскольку граница неравенства x < 2 не принадлежит множеству его решений.

На письме открытый числовой луч, заданный неравенством x < a, обозначается следующим образом:

( −∞ ; a )

Запишем ответ к неравенству x < 2 с помощью обозначения открытого числового луча:

x ∈  ( −∞ ; 2 )

В этом выражении говорится, что все числа от минус бесконечности до 2, являются решениями неравенства x < 2. Граница 2 не принадлежит множеству решений, поскольку неравенство x < 2 является строгим.

Отрезок

Отрезком называют числовой промежуток, который задаётся двойным неравенством a ≤ x ≤ b, где a и b — границы данного неравенства, x — решение неравенства.

Пусть a = 2, b = 8. Тогда неравенство a ≤ x ≤ b примет вид 2 ≤ ≤ 8. Решениями неравенства 2 ≤ ≤ 8 являются все числа, которые больше 2 и меньше 8. При этом границы неравенства 2 и 8 принадлежат множеству его решений, поскольку неравенство 2 ≤ ≤ 8 является нестрогим.

Изобразим отрезок, заданный двойным неравенством 2 ≤ ≤ 8 на координатной прямой. Для этого отметим на ней точки с координатами 2 и 8, а располагающуюся между ними область выделим штрихами:

Здесь точки 2 и 8 соответствуют границам неравенства 2 ≤ ≤ 8, а выделенная штрихами область соответствует множеству значений x, которые являются решениями неравенства 2 ≤ ≤ 8. Точки 2 и 8, являющиеся границами отрезка, изображены в виде закрашенных кружков, поскольку границы неравенства 2 ≤ ≤ 8 принадлежат множеству его решений.

На письме отрезок, заданный неравенством a ≤ x ≤ b обозначается следующим образом:

[ a ; b ]

Квадратные скобки с обеих сторон указывают на то, что границы отрезка принадлежат ему. Запишем ответ к неравенству 2 ≤ ≤ 8 с помощью этого обозначения:

x ∈  [ 2 ; 8 ]

В этом выражении говорится, что все числа от 2 до 8 включительно, являются решениями неравенства 2 ≤ ≤ 8.

Интервал

Интервалом называют числовой промежуток, который задаётся двойным неравенством a < x < b, где a и b — границы данного неравенства, x — решение неравенства.

Пусть a = 2, b = 8. Тогда неравенство a < x < b примет вид 2 < < 8. Решениями этого двойного неравенства являются все числа, которые больше 2 и меньше 8, исключая числа 2 и 8.

Изобразим интервал на координатной прямой:

Здесь точки 2 и 8 соответствуют границам неравенства 2 < < 8, а выделенная штрихами область соответствует множеству значений x, которые являются решениями неравенства 2 < < 8. Точки 2 и 8, являющиеся границами интервала, изображены в виде пустых кружков, поскольку границы неравенства 2 < < 8 не принадлежат множеству его решений.

На письме интервал, заданный неравенством a < x < b, обозначается следующим образом:

( a ; b )

Круглые скобки с обеих сторон указывают на то, что границы интервала не принадлежат ему. Запишем ответ к неравенству 2 < < 8 с помощью этого обозначения:

x ∈  ( 2 ; 8 )

В этом выражении говорится, что все числа от 2 до 8, исключая числа 2 и 8, являются решениями неравенства 2 < < 8.

Полуинтервал

Полуинтервалом называют числовой промежуток, который задаётся неравенством a ≤ x < b, где a и b — границы данного неравенства, x — решение неравенства.

Полуинтервалом также называют числовой промежуток, который задаётся неравенством a < x ≤ b.

Одна из границ полуинтервала принадлежит ему. Отсюда и название этого числового промежутка.

В ситуации с полуинтервалом a ≤ x < b ему (полуинтервалу) принадлежит левая граница.

А в ситуации с полуинтервалом a < x ≤ b ему принадлежит правая граница.

Пусть = 2, = 8. Тогда неравенство a ≤ x < b примет вид 2 ≤ x < 8. Решениями этого двойного неравенства являются все числа, которые больше 2 и меньше 8, включая число 2, но исключая число 8.

Изобразим полуинтервал 2 ≤ x < 8 на координатной прямой:

Здесь точки 2 и 8 соответствуют границам неравенства 2 ≤ x < 8, а выделенная штрихами область соответствует множеству значений x, которые являются решениями неравенства 2 ≤ x < 8.

Точка 2, являющаяся левой границей полуинтервала, изображена в виде закрашенного кружка, поскольку левая граница неравенства 2 ≤ x < 8 принадлежит множеству его решений.

А точка 8, являющаяся правой границей полуинтервала, изображена в виде пустого кружка, поскольку правая граница неравенства 2 ≤ x < 8 не принадлежит множеству его решений.

На письме полуинтервал, заданный неравенством a ≤ x < b, обозначается следующим образом:

a ; b )

Видно, что с одной стороны граница обрамлена квадратной скобкой, а с другой круглой. Это связано с тем, что одна граница полуинтервала принадлежит ему, а другая нет. Запишем ответ к неравенству 2 ≤ x < 8 с помощью этого обозначения:

x ∈  [ 2 ; 8 )

В этом выражении говорится, что все числа от 2 до 8, включая число 2, но исключая число 8, являются решениями неравенства 2 ≤ x < 8.

Аналогично на координатной прямой можно изобразить полуинтервал, заданный неравенством a < x ≤ b. Пусть = 2, = 8. Тогда неравенство a < x ≤ b примет вид 2 < ≤ 8. Решениями этого двойного неравенства являются все числа, которые больше 2 и меньше 8, исключая число 2, но включая число 8.

Изобразим полуинтервал 2 < ≤ 8 на координатной прямой:

Здесь точки 2 и 8 соответствуют границам неравенства 2 < ≤ 8, а выделенная штрихами область соответствует множеству значений x, которые являются решениями неравенства 2 < ≤ 8.

Точка 2, являющаяся левой границей полуинтервала, изображена в виде пустого кружка, поскольку левая граница неравенства 2 < ≤ 8 не принадлежит множеству его решений.

А точка 8, являющаяся правой границей полуинтервала, изображена в виде закрашенного кружка, поскольку правая граница неравенства 2 < ≤ 8 принадлежит множеству его решений.

На письме полуинтервал, заданный неравенством a < x ≤ b, обозначается так: ( a ; b ]. Запишем ответ к неравенству 2 < ≤ 8 с помощью этого обозначения:

x ∈  ( 2 ; 8 ]

В этом выражении говорится, что все числа от 2 до 8, исключая число 2, но включая число 8, являются решениями неравенства 2 < ≤ 8.


Изображение числовых промежутков на координатной прямой

Числовой промежуток может быть задан с помощью неравенства или с помощью обозначения (круглых или квадратных скобок). В обоих случаях нужно суметь изобразить этот числовой промежуток на координатной прямой. Рассмотрим несколько примеров.

Пример 1. Изобразить числовой промежуток, заданный неравенством > 5

Вспоминаем, что неравенством вида a задаётся открытый числовой луч. В данном случае переменная a равна 5. Неравенство > 5 строгое, поэтому граница 5 будет изображаться в виде пустого кружкá. Нас интересуют все значения x, которые больше 5, поэтому вся область справа будет выделена штрихами:


Пример 2. Изобразить числовой промежуток (5; +∞) на координатной прямой

Это тот же числовой промежуток, который мы изобразили в предыдущем примере. Но в этот раз он задан не с помощью неравенства, а с помощью обозначения числового промежутка.

Граница 5 обрамлена круглой скобкой, значит она не принадлежит промежутку. Соответственно, кружок остаётся пустым.

Символ +∞ указывает, что нас интересуют все числа, которые больше 5. Соответственно, вся область справа от границы 5 выделяется штрихами:


Пример 3. Изобразить числовой промежуток (−5; 1) на координатной прямой.

Круглыми скобками с обеих сторон обозначаются интервалы. Границы интервала не принадлежат ему, поэтому границы −5 и 1 будут изображаться на координатной прямой в виде пустых кружков. Вся область между ними будет выделена штрихами:


Пример 4. Изобразить числовой промежуток, заданный неравенством −5 < x < 1

Это тот же числовой промежуток, который мы изобразили в предыдущем примере. Но в этот раз он задан не с помощью обозначения промежутка, а с помощью двойного неравенства.

Неравенством вида a < x < b, задаётся интервал. В данном случае переменная a равна −5, а переменная b равна единице. Неравенство −5 < x < 1 строгое, поэтому границы −5 и 1 будут изображаться в виде пустых кружка. Нас интересуют все значения x, которые больше −5, но меньше единицы, поэтому вся область между точками −5 и 1 будет выделена штрихами:


Пример 5. Изобразить на координатной прямой числовые промежутки [−1; 2) и [2; 5]

В этот раз изобразим на координатной прямой сразу два промежутка. Промежуток [−1; 2) является полуинтервалом, промежуток [2; 5] — отрезком.

У полуинтервала [−1; 2) левая граница принадлежит ему, а правая нет.

А у отрезка [2; 5] обе границы принадлежат ему.

Чтобы хорошо увидеть промежутки [−1; 2) и [2; 5], первый можно изобразить на верхней области, а второй на нижней. Так и поступим:

Граница 2 закрашена потому что она входит в промежуток [2; 5].


Пример 6. Изобразить на координатной прямой числовые промежутки [-1; 2) и (2; 5]

Квадратной скобкой с одной стороны и круглой с другой обозначаются полуинтервалы. Одна из границ полуинтервала принадлежат ему, а другая нет.

В случае с полуинтервалом [-1; 2) левая граница будет принадлежать ему, а правая нет. Значит левая граница будет изображаться в виде закрашенного кружка. Правая же граница будет изображаться в виде пустого кружка.

А в случае с полуинтервалом (2; 5] ему будет принадлежать только правая граница, а левая нет. Значит левая граница будет изображаться в виде пустого кружка. Правая же граница будет изображаться в виде закрашенного кружка.

Изобразим промежуток [-1; 2) на верхней области координатной прямой, а промежуток (2; 5] — на нижней:


Примеры решения неравенств

Неравенство, которое путём тождественных преобразований можно привести к виду ax > b (или к виду ax < b), будем называть линейным неравенством с одной переменной.

В линейном неравенстве ax > b, x — это переменная, значения которой нужно найти, а — коэффициент этой переменной, b — граница неравенства, которая в зависимости от знака неравенства может принадлежать множеству его решений либо не принадлежать ему.

Например, неравенство 2> 4 является неравенством вида ax > b. В нём роль переменной a играет число 2, роль переменной b (границы неравенства) играет число 4.

Неравенство 2> 4 можно сделать ещё проще. Если мы разделим обе его части на 2, то получим неравенство > 2

Получившееся неравенство > 2 также является неравенством вида ax > b, то есть линейным неравенством с одной переменной. В этом неравенстве роль переменной a играет единица. Ранее мы говорили, что коэффициент 1 не записывают. Роль переменной b играет число 2.

Отталкиваясь от этих сведений, попробуем решить несколько простых неравенств. В ходе решения мы будем выполнять элементарные тождественные преобразования с целью получить неравенство вида ax > b

Пример 1. Решить неравенство − 7 < 0

Прибавим к обеим частям неравенства число 7

− 7 + 7 < 0 + 7

В левой части останется x, а правая часть станет равна 7

< 7

Путём элементарных преобразований мы привели неравенство − 7 < 0 к равносильному неравенству < 7. Решениями неравенства < 7 являются все числа, которые меньше 7. Граница 7 не принадлежит множеству решений, поскольку неравенство строгое.

Когда неравенство приведено к виду x < a (или x > a), его можно считать уже решённым. Наше неравенство − 7 < 0 тоже приведено к такому виду, а именно к виду < 7. Но в большинстве школ требуют, чтобы ответ был записан с помощью числового промежутка и проиллюстрирован на координатной прямой.

Запишем ответ с помощью числового промежутка. В данном случае ответом будет открытый числовой луч (вспоминаем, что числовой луч задаётся неравенством x < a и обозначается как ( −∞ ; a)

x ∈  ( −∞ ; 7 )

На координатной прямой граница 7 будет изображаться в виде пустого кружка, а вся область, находящаяся слева от границы, будет выделена штрихами:

Для проверки возьмём любое число из промежутка ( −∞ ; 7 ) и подставим его в неравенство < 7 вместо переменной x. Возьмём, например, число 2

2 < 7

Получилось верное числовое неравенство, значит и решение верное. Возьмём ещё какое-нибудь число, например, число 4

4 < 7

Получилось верное числовое неравенство. Значит решение верное.

А поскольку неравенство < 7 равносильно исходному неравенству x − 7 < 0, то решения неравенства < 7 будут совпадать с решениями неравенства x − 7 < 0. Подставим те же тестовые значения 2 и 4 в неравенство x − 7 < 0

2 − 7 < 0

−5 < 0 — Верное неравенство

4 − 7 < 0

−3 < 0 Верное неравенство


Пример 2. Решить неравенство −4x < −16

Разделим обе части неравенства на −4. Не забываем, что при делении обеих частей неравенства на отрицательное число, знак неравенства меняется на противоположный:

Мы привели неравенство −4x < −16 к равносильному неравенству > 4. Решениями неравенства > 4 будут все числа, которые больше 4. Граница 4 не принадлежит множеству решений, поскольку неравенство строгое.

Изобразим множество решений неравенства > 4 на координатной прямой и запишем ответ в виде числового промежутка:


Пример 3. Решить неравенство 3y + 1 > 1 + 6y

Перенесём 6y из правой части в левую часть, изменив знак. А 1 из левой части перенесем в правую часть, опять же изменив знак:

3− 6y> 1 − 1

Приведём подобные слагаемые:

−3y > 0

Разделим обе части на −3. Не забываем, что при делении обеих частей неравенства на отрицательное число, знак неравенства меняется на противоположный:

Решениями неравенства < 0 являются все числа, меньшие нуля. Изобразим множество решений неравенства < 0 на координатной прямой и запишем ответ в виде числового промежутка:


Пример 4. Решить неравенство 5(− 1) + 7 ≤ 1 − 3(+ 2)

Раскроем скобки в обеих частях неравенства:

Перенесем −3x из правой части в левую часть, изменив знак. Члены −5 и 7 из левой части перенесем в правую часть, опять же изменив знаки:

Приведем подобные слагаемые:

Разделим обе части получившегося неравенства на 8

Решениями неравенства  являются все числа, которые меньше . Граница принадлежит множеству решений, поскольку неравенство  является нестрогим.

Изобразим множество решений неравенства  на координатной прямой и запишем ответ в виде числового промежутка:

 


Пример 5. Решить неравенство 

Умножим обе части неравенства на 2. Это позволит избавиться от дроби в левой части:

Теперь перенесем 5 из левой части в правую часть, изменив знак:

После приведения подобных слагаемых, получим неравенство 6> 1. Разделим обе части этого неравенства на 6. Тогда получим:

Решениями неравенства  являются все числа, которые больше . Граница  не принадлежит множеству решений, поскольку неравенство  является строгим.

Изобразим множество решений неравенства  на координатной прямой и запишем ответ в виде числового промежутка:


Пример 6. Решить неравенство 

Умножим обе части на 6

После приведения подобных слагаемых, получим неравенство 5< 30. Разделим обе части этого неравенства на 5

Решениями неравенства < 6 являются все числа, которые меньше 6. Граница 6 не принадлежит множеству решений, поскольку неравенство является < 6 строгим.

Изобразим множество решений неравенства < 6 на координатной прямой и запишем ответ в виде числового промежутка:


Пример 7. Решить неравенство 

Умножим обе части неравенства на 10

В получившемся неравенстве раскроем скобки в левой части:

Перенесем члены без x в правую часть

Приведем подобные слагаемые в обеих частях:

Разделим обе части получившегося неравенства на 10

Решениями неравенства ≤ 3,5 являются все числа, которые меньше 3,5. Граница 3,5 принадлежит множеству решений, поскольку неравенство является ≤ 3,5 нестрогим.

Изобразим множество решений неравенства ≤ 3,5 на координатной прямой и запишем ответ в виде числового промежутка:


Пример 8. Решить неравенство 4 < 4< 20

Чтобы решить такое неравенство, нужно переменную x освободить от коэффициента 4. Тогда мы сможем сказать в каком промежутке находится решение данного неравенства.

Чтобы освободить переменную x от коэффициента, можно разделить член 4x на 4. Но правило в неравенствах таково, что если мы делим член неравенства на какое-нибудь число, то тоже самое надо сделать и с остальными членами, входящими в данное неравенство. В нашем случае на 4 нужно разделить все три члена неравенства 4 < 4< 20

Решениями неравенства 1 < < 5 являются все числа, которые больше 1 и меньше 5. Границы 1 и 5 не принадлежат множеству решений, поскольку неравенство 1 < < 5 является строгим.

Изобразим множество решений неравенства 1 < < 5 на координатной прямой и запишем ответ в виде числового промежутка:


Пример 9. Решить неравенство −1 ≤ −2≤ 0

Разделим все члены неравенства на −2

Получили неравенство 0,5 ≥ ≥ 0. Двойное неравенство желательно записывать так, чтобы меньший член располагался слева, а больший справа. Поэтому перепишем наше неравенство следующим образом:

0 ≤ ≤ 0,5

Решениями неравенства 0 ≤ ≤ 0,5 являются все числа, которые больше 0 и меньше 0,5. Границы 0 и 0,5 принадлежат множеству решений, поскольку неравенство 0 ≤ ≤ 0,5 является нестрогим.

Изобразим множество решений неравенства 0 ≤ ≤ 0,5 на координатной прямой и запишем ответ в виде числового промежутка:


Пример 10. Решить неравенство 

Умножим обе неравенства на 12

Раскроем скобки в получившемся неравенстве и приведем подобные слагаемые:

Разделим обе части получившегося неравенства на 2

Решениями неравенства ≤ −0,5 являются все числа, которые меньше −0,5. Граница −0,5 принадлежит множеству решений, поскольку неравенство ≤ −0,5 является нестрогим.

Изобразим множество решений неравенства ≤ −0,5 на координатной прямой и запишем ответ в виде числового промежутка:


Пример 11. Решить неравенство 

Умножим все части неравенства на 3

Теперь из каждой части получившегося неравенства вычтем 6

Каждую часть получившегося неравенства разделим на −1. Не забываем, что при делении всех частей неравенства на отрицательное число, знак неравенства меняется на противоположный:

Решениями неравенства 3 ≤ a ≤ 9 являются все числа, которые больше 3 и меньше 9. Границы 3 и 9 принадлежат множеству решений, поскольку неравенство 3 ≤ a ≤ 9 является нестрогим.

Изобразим множество решений неравенства 3 ≤ a ≤ 9 на координатной прямой и запишем ответ в виде числового промежутка:


Когда решений нет

Существуют неравенства, которые не имеют решений. Таковым, например, является неравенство 6> 2(3+ 1). В процессе решения этого неравенства мы придём к тому, что знак неравенства > не оправдает своего местоположения. Давайте посмотрим, как это выглядит.

Раскроем скобки в правой части данного неравенство, получим 6> 6+ 2. Перенесем 6x из правой части в левую часть, изменив знак, получим 6− 6> 2. Приводим подобные слагаемые и получаем неравенство 0 > 2, которое не является верным.

Для наилучшего понимания, перепишем приведение подобных слагаемых в левой части следующим образом:

Получили неравенство 0> 2. В левой части располагается произведение, которое будет равно нулю при любом x. А ноль не может быть больше, чем число 2. Значит неравенство 0> 2 не имеет решений.

А если не имеет решений приведённое равносильное неравенство 0> 2, то не имеет решений и исходное неравенство 6> 2(3+ 1).


Пример 2. Решить неравенство 

Умножим обе части неравенства на 3

В получившемся неравенстве перенесем член 12x из правой части в левую часть, изменив знак. Затем приведём подобные слагаемые:

Правая часть получившегося неравенства при любом x будет равна нулю. А ноль не меньше, чем −8. Значит неравенство 0< −8 не имеет решений.

А если не имеет решений приведённое равносильное неравенство 0< −8, то не имеет решений и исходное неравенство .

Ответ: решений нет.


Когда решений бесконечно много

Существуют неравенства, имеющие бесчисленное множество решений. Такие неравенства становятся верными при любом x.

Пример 1. Решить неравенство 5(3− 9) < 15x

Раскроем скобки в правой части неравенства:

Перенесём 15x из правой части в левую часть, изменив знак:

Приведем подобные слагаемые в левой части:

Получили неравенство 0x < 45. В левой части располагается произведение, которое будет равно нулю при любом x. А ноль меньше, чем 45. Значит решением неравенства 0x < 45 является любое число.

А если приведённое равносильное неравенство 0x < 45 имеет бесчисленное множество решений, то и исходное неравенство 5(3− 9) < 15x имеет те же решения.

Ответ можно записать в виде числового промежутка:

x ∈ ( −∞; +∞ )

В этом выражении говорится, что решениями неравенства 5(3− 9) < 15x являются все числа от минус бесконечности до плюс бесконечности.


Пример 2. Решить неравенство: 31(2+ 1) − 12> 50x

Раскроем скобки в левой части неравенства:

Перенесём 50x из правой части в левую часть, изменив знак. А член 31 из левой части перенесём в правую часть, опять же изменив знак:

Приведём подобные слагаемые:

Получили неравенство 0x > −31. В левой части располагается произведение, которое будет равно нулю при любом x. А ноль больше, чем −31. Значит решением неравенства 0x < −31 является любое число.

А если приведённое равносильное неравенство 0x > −31 имеет бесчисленное множество решений, то и исходное неравенство 31(2+ 1) − 12> 50x имеет те же решения.

Запишем ответ в виде числового промежутка:

x ∈ ( −∞; +∞ )


Задания для самостоятельного решения

Задание 1. Решите неравенство:

Задание 2. Решите неравенство:

Задание 3. Решите неравенство:

Задание 4. Решите неравенство:

Задание 5. Решите неравенство:

Задание 6. Решите неравенство:

Задание 7. Решите неравенство:

Задание 8. Решите неравенство:

Задание 9. Решите неравенство:

Задание 10. Решите неравенство:

Задание 11. Решите неравенство:

Задание 12. Решите неравенство:


Понравился урок?
Вступай в нашу новую группу Вконтакте и начни получать уведомления о новых уроках

Возникло желание поддержать проект?
Используй кнопку ниже

Навигация по записям

понятие, ОДЗ, равносильность (Автор Колчанов А.В.)

Алгебраические неравенства. Подготовка к ЕГЭ.

Универсальный метод решения алгебраических неравенств заключается в приведении их с помощью равносильных преобразований к системам или совокупностям легко решаемых рациональных неравенств или уравнений. Этот метод школьники осваивают, начиная с 9-го класса. В 10 – 11 классах средней школы, рассматривая кроме алгебраических еще тригонометрические, показательные и логарифмические уравнения и неравенства, как правило, с помощью замен или других рассуждений удается решение свести к исследованию равносильных систем или совокупностей простейших уравнений и неравенств.

Понятия неравенства с переменной и его решений

Если два выражения с переменной соединить одним из знаков >, <, <, то получаем неравен­ство с переменной.

Аналогично уравнению, неравенство с переменной (например, со зна­ком >) чаще всего понимают как аналитическую запись задачи о нахож­дении тех значений аргументов, при которых значение одной из заданных функций больше, чем значение другой заданной функции. Поэтому в общем виде неравенство с одной переменной х (например, для случаев «больше») записывают так: f(x)>g (*).

Напомним, что решением неравенства называется значение переменной, которое обращает это неравенство в верное числовое неравенство.

Решить неравенство — значит найти все его решения (и обосновать, что других решений нет) или доказать, что решений нет.

Например, решениями неравенства Зх < 6 являются все значения х < 2, для неравенства х2 > -1 решениями являются все действительные числа (R), а неравенство х2 <-1 не имеет решений, поскольку значение х2 не может быть отрицательным числом.

Область допустимых значений (ОДЗ)

Область допустимых значений (ОДЗ) неравенства определяется аналогично ОДЗ уравнения. Если задано неравенство f (х) > g(x), то общая область определения функций f(x) и g(x) называется областью допустимых значений этого неравенства (иногда используются также термины «область определения неравенства* или «множество допустимых значений неравенства*).

Например, для неравенства х2 < х областью допустимых значений являются все действительные числа (это можно записать, например, так: ОДЗ: R), поскольку функции f(x) = х2 и g(x) = х имеют области определения R.

Понятно, что каждое решение заданного неравенства входит как в область определения функции f(x), так и в область определения функции g(x) (иначе мы не сможем получить верное числовое неравенство). Таким образом, каждое решение неравенства обязательно входит в ОДЗ этого неравенства. Это позволяет в некоторых случаях применить анализ ОДЗ неравенства для его решения.

Равносильные неравенства

С понятием равносильности неравенств вы знакомы еще из курса алгебры 9 класса. Как и для случая равносильных уравнений, равносильность неравенств мы будем рассматривать на опреде­ленном множестве.

Два неравенства называются равносильными на некотором мно­жестве, если на этом множестве они имеют одни и те же реше­ния, то есть каждое решение первого неравенства является реше­нием второго и, наоборот, каждое решение второго неравенства является решением первого.

Договоримся, что в дальнейшем все равносильные преобразования не­равенств будем выполнять на ОДЗ заданного неравенства. Укажем, что в том случае, когда ОДЗ заданного неравенства является множество всех действительных чисел, мы не всегда будем его записывать (как не записы­вали ОДЗ при решении линейных или квадратных неравенств). И в других случаях главное — не записать ОДЗ при решении неравенства, а действи­тельно учесть ее при выполнении равносильных преобразований заданного неравенства.

Общие ориентиры выполнения равносильных преобразований неравенств аналогичны соответствующим ориентирам выполнения равносильных пре­образований уравнений.

Как указывалось выше, выполняя равносильные преобразования нера­венств, необходимо учитывать ОДЗ заданного неравенства — это и есть первый ориентир для выполнения равносильных преобразований нера­венств.

По определению равносильности неравенств необходимо обеспечить, что­бы каждое решение первого неравенства было решением второго, и наобо­рот, каждое решение второго неравенства было решением первого. Для это­го достаточно обеспечить сохранение верного неравенства на каждом шаге решения не только при прямых, но и при обратных преобразованиях. Это и есть второй ориентир для решения неравенств с помощью равносиль­ных преобразований. Действительно, каждое решение неравенства обращает его в верное числовое неравенство, и если верное неравенство сохраняется, то решение каждого из неравенств будет также и решением другого, таким образом, неравенства будут равносильны.

Например, чтобы решить с помощью равносильных преобразований не­равенство

достаточно учесть его ОДЗ: х + 1  не не равно 0 и условие положительности дроби (дробь будет положительной тогда и только тогда, когда числитель и зна­менатель дроби имеют одинаковые знаки), а также учесть, что на ОДЗ все необходимые преобразования можно выполнить как в прямом, так и в об­ратном направлении с сохранением верного неравенства.

Кроме выделенных общих ориентиров, для выполнения равносильных преобразований неравенств можно также пользоваться специальными теоремами о равносильности. В связи с уточнением определения равносильности неравенств обобщим также формулировки простейших теорем о равносильности неравенств, известных из курса алгебры 9 класса.

1. Если из одной части неравенства перенести в другую часть слагаемые с противоположным знаком, то получим неравенство, равносильное заданному (на любом множестве).

2. Если обе части неравенства умножить или разделить на одно и то же положительное число (или на одну и ту же функцию, которая определена и положительна на ОДЗ заданного неравенства), не изменяя знак неравенства, то получим неравенство, равносильное заданному (на ОДЗ заданного).

3. Если обе части неравенства умножить или разделить на одно и то же отрицательное число (или на одну и ту же функцию, которая определена и отрицательна на ОДЗ заданного неравенства) и изменить знак неравенства на противоположный, то получим неравенство, равносильное заданному (на ОДЗ заданного).

Обоснование этих теорем проводится с использованием основных свойств числовых неравенств и полностью аналогично обоснованию ориентиров для равносильных преобразований заданного неравенства.

Замечание. Для обозначения перехода от заданного неравенства к неравенству, равносильному ему, можно применять специальный значок <=>, но его использование при оформлении решений не является обязательным.

Числовые неравенства и их свойства

С неравенствами мы познакомились  в школе, где применяем числовые неравенства. В данной статье рассмотрим свойства числовых неравенств, не которых строятся принципы работы с ними.

Свойства неравенств аналогичны свойствам числовых неравенств. Будут рассмотрены свойства, его обоснования, приведем примеры.

Числовые неравенства: определение, примеры

При введении понятия неравенства имеем, что их определение производится по виду записи.  Имеются алгебраические выражения, которые имеют знаки ≠, <, >, ≤ , ≥. Дадим определение.

Определение 1

Числовым неравенством называют неравенство, в записи которого обе стороны имеют числа и числовые выражения.

Числовые неравенства рассматриваем еще в школе после изучения натуральных чисел. Такие операции сравнения изучаются поэтапно. Первоначальные имею вид 1<5, 5+7>3. После чего правила дополняются, а неравенства усложняются, тогда получаем неравенства вида 523>5,1(2), ln 0.73-172<0.

Свойства числовых неравенств

Чтобы правильно работать с неравенствами, необходимо использовать свойства числовых неравенств. Они идут из понятия неравенства. Такое понятие задается при помощи утверждения, которое обозначается как «больше» или «меньше».

Определение 2
  • число a больше b, когда  разность a-b – положительное число;
  • число a меньше b, когда разность a-b – отрицательное число;
  • число a равно b, когда разность a-bравняется нулю.

Определение используется при решении неравенств с отношениями «меньше или равно», «больше или равно». Получаем, что

Определение 3
  • a больше или равно b, когда a-b является неотрицательным числом;
  • a меньше или равно b, когда a-b является неположительным числом.

Определения будут использованы при доказательствах свойств числовых неравенств.

Основные свойства

Рассмотрим 3 основные неравенства. Использование знаков < и > характерно при свойствах:

Определение 4
  • антирефлексивности, которое говорит о том, что любое число a из неравенств a<a и a>a считается неверным. Известно, что для любого a имеет место быть равенство a−a=0, отсюда получаем, что а=а. Значит, a<a и a>a неверно. Например, 3<3 и -41415>-41415 являются неверными.
  • ассиметричности. Когда числа a и b являются такими, что a<b, то b>a, и если a>b, то b<a.  Используя определение отношений «больше», «меньше» обоснуем его. Так как в первой части имеем, что a<b, тогда  a−b является отрицательным числом. А b−a=−(a−b) положительное число, потому как число противоположно отрицательному числу a−b. Отсюда следует, что b>a. Аналогичным образом доказывается  и вторая его часть.
Пример 1

Например, при заданном неравенстве 5<11 имеем, что 11>5, значит его числовое неравенство −0,27>−1,3 перепишется  в виде −1,3<−0,27.

Перед тем, как перейти к следующему свойству, заметим, что при помощи ассиметричности можно читать неравенство справа налево и наоборот. Таким образом, числовое неравенство можно изменять  и менять местами.

Определение 5
  • транзитивности.  Когда числа a, b, c соответствуют условию a<b и b<c, тогда  a<c, и если a>b и b>c, тогда a>c.  
Доказательство 1

Первое утверждение можно доказать. Условие a<b и b<c означает, что a−b и b−c являются отрицательными,  а разность а-с представляется  в виде (a−b)+(b−c), что является отрицательным числом, потому как имеем сумму двух отрицательных a−b и b−c. Отсюда получаем, что а-с является отрицательным числом, а значит, что a<c. Что и требовалось доказать.

Аналогичным образом доказывается вторая часть со свойством транизитивности.

Пример 2

Разобранное свойство рассматриваем на примере неравенств −1<5 и 5<8. Отсюда имеем, что −1<8. Аналогичным образом из неравенств 12>18 и 18>132 следует, что 12>132.

Числовые неравенства, которые записываются  с помощью нестрогих знаков неравенства, обладают свойством рефлексивности, потому как a≤a и a≥a  могут иметь случай равенства а=а. им присуща ассиметричность и транзитивность.

Определение 6

Неравенства, имеющие  в записи знаки ≤ и≥, имеют свойства:

  • рефлексивности a≥a и a≤a считаются  верными неравенствами;
  • антисимметричности, когда a≤b, тогда b≥a, и если a≥b, тогда b≤a.
  • транзитивности, когда a≤b и b≤c, тогда a≤c, а также, если a≥b и b≥c, то тогда a≥c.

Доказательство производится  аналогичным образом.

Другие важные свойства числовых неравенств

Для дополнения основных свойств неравенств используются результаты, которые имеют практическое значение. Применяется принцип метода оценка значений выражений, на которых и базируются принципы решения неравенств.

Данный пункт раскрывает свойства неравенств для одного знака строгого неарвенства. Аналогично производится для нестрогих. Рассмотрим на примере, сформулировав неравенство если a<b и c являются любыми числами, то a+c<b+c. Справедливыми окажутся свойства:

  • если a>b, то a+c>b+c;
  • если a≤b, то a+c≤b+c;
  • если a≥b, то a+c≥b+c.

Для удобного представления дадим соответствующее утверждение, которое записывается  и приводятся доказательства, показываются примеры использования.

Определение 7

Прибавление или вычисления числа к обеим сторонам. Иначе говоря, когда a и b соответствуют неравенству a<b, тогда для любого такого числа имеет смысл неравенство вида a+c<b+c.

Доказательство 2

Чтобы доказать это, необходимо, чтобы уравнение соответствовало условию a<b. Тогда (a+c)−(b+c)=a+c−b−c=a−b. Из условия a<b получим, что a−b<0. Значит, (a+c)−(b+c)<0, откуда a+c<b+c. Множество действительных числе могут быть изменены  с помощью прибавления противоположного числа –с.

Пример 3

К примеру, если обе части неравенства 7>3 увеличиваем на 15, тогда получаем, что 7+15>3+15. Это равно 22>18.

Нужна помощь преподавателя?

Опиши задание — и наши эксперты тебе помогут!

Описать задание Определение 8

Когда обе части неравенства умножить или разделить на одно и то же число c, получим верное неравенство.  Если взять число c отрицательным, то знак поменяется на противоположный. Иначе это выглядит так: для  a и b неравенство выполняется, когда  a<b и c являются положительными числами, то a·c<b·c, а если v является отрицательным  числом, тогда a·c>b·c.

Доказательство 3

Когда имеется случай  c>0, необходимо составить разность левой и правой частей неравенства. Тогда получаем, что a·c−b·c=(a−b)·c. Из условия a<b, то a−b<0, а c>0, тогда произведение (a−b)·c будет отрицательным. Отсюда следует, что a·c−b·c<0, где a·c<b·c. Другая часть доказывается аналогичным образом.

При доказательстве деление на целое число можно заменить умножением на обратное заданному, то есть 1c.  Рассмотрим пример свойства на определенных числах.

Пример 4

Разрешено обе части неравенства 4<6 умножаем на положительное 0,5, тогда получим неравенство вида −4·0,5<6·0,5, где −2<3. Когда обе части делим на -4, то необходимо изменить знак неравенства на противоположный . отсюда имеем, что неравенство примет вид −8:(−4)≥12:(−4), где 2≥−3.

Теперь сформулируем вытекающие два результата, которые используются при решении неравенств:

  • Следствие 1. При смене знаков частей числового неравенства меняется  сам знак неравенства на противоположный, как a<b, как −a>−b. Это соответствует правилу умножения обеих частей на -1. Оно применимо для перехода. Например, −6<−2, то 6>2.
  • Следствие 2. При замене обратными числами частей числового неравенства на противоположный, меняется и его знак, причем неравенство останется верным. Отсюда имеем, что a и b являются положительными числами,  a<b, 1a>1b.

При делении обеих частей неравенства a<b разрешается на число a·b. Данное свойство используется при верном неравенстве 5>32  имеем, что 15<23.  При отрицательных a и b c условием, что a<b , неравенство 1a>1b может получиться неверным.

Пример 5

Например, −2<3, однако, -12>13 являются неверным равенством.

Все пункты объединяет то, что действия над частями неравенства  дают верное неравенство на выходе. Рассмотрим свойства, где изначально имеется несколько числовых неравенств, а его результат получим при сложении  или умножении его частей.

Определение 9

Когда числа a, b, c, d справедливы для неравенств a<b и c<d, тогда верным считается a+c<b+d. Свойство можно формировать таким образом: почленно складывать числа частей неравенства.

Доказательство 4

Докажем, что (a+c)−(b+d)  является отрицательным числом, тогда получим, что a+c<b+d. Из условия имеем, что a<b и c<d. Выше доказанное свойство позволяет прибавлять к обеим частям одинаковое число. Тогда увеличим неравенство a<b на число b, при c<d, получим неравенства вида a+c<b+c и b+c<b+d. Полученное неравенство говорит о том, что ему присуще свойство транзитивности.

Свойство применяется для почленного сложения трех, четырех и более числовых неравенств. Числам a1, a2, …, an и b1, b2, …, bn справедливы неравенства a1<b1, a2<b2, …, an<bn , можно доказать метод математической индукции , получив a1+a2+…+an<b1+b2+…+bn.

Пример 6

Например, при данных трех числовых неравенствах одного знака −5<−2, −1<12 и 3<4. Свойство позволяет определять  то, что −5+(−1)+3<−2+12+4 является верным.

Определение 10

Почленное умножение обеих частей дает в результате положительное число. При a<b и c<d, где a, b, c и d являются положительными числами, тогда неравенство вида a·c<b·d считается справедливым.

Доказательство 5

Чтобы доказать это, необходимо обе части неравенства a<b умножить на число с, а обе части c<d на b. В итоге получим, что неравенства a·c<b·c и b·c<b·d верные, откуда получим свойство транизитивности a·c<b·d.

Это свойство считается справедливым для количества чисел, на которые необходимо умножить обе части неравенства. Тогда a1, a2, …, an и b1, b2, …, bnявляются положительные числами, где a1<b1, a2<b2, …, an<bn, то a1·a2·…·an<b1·b2·…·bn.

Заметим, что при записи неравенств имеются неположительные числа, тогда их почленное умножение приводит к неверным неравенствам.

Пример 7

К примеру, неравенство 1<3 и −5<−4 являются верными, а почленное их умножение даст результат в виде 1·(−5)<3·(−4), считается, что  −5<−12 это является неверным неравенством.

Следствие: Почленное умножение неравенств a<b с положительными с a и b, причем получается an<bn.

Свойства числовых неравенств

Рассмотрим ниже свойства числовых неравенств.

  1. a<a, a>a — неверные неравенства,
    a≤a, a≥a- верные неравенства.
  2. Если a<b, то b>a — антисимметричность.
  3. Если a<b и b<c  то a<c — транзитивность.
  4. Если a<b и c — любоое число, то a+с<b+c.
  5. Если a<b и c — положительное число, то a·c<b·c,
    Если a<b и c — отрицательное число, то a·c>b·c.

Следствие 1: если a<b, то -a>-b.

Следствие 2: если a и b — положительные числа и a<b, то 1a>1b.

  1. Если a1<b1, a2<b2,…, an<bn, то a1+a2+…+an<b1+b2+…+bn.
  2. Если a1, a2,…, an, b1, b2,…,bn- положительные числа и a1<b1, a2<b2,…, an<bn, то a1·a2·…·an<b1·b2·…bn.

Cледствие 1: если  a<b, a и  b — положительные числа, то an<bn.

Найди запись, в которой число записано в виде разрядных слагаемых


Вариант 002

  1. Найди запись, в которой число записано в виде разрядных слагаемых;

А) 89063 = 89000 + 63

В) 320502 = 3000000 + 20+500 +2

С) 568041 = 500000 + 60000 + 8000 + 40 + 1

D) 506812 = 506000 + 812


  1. Определи тип данного выражения;

(400 + 60 +9)*2+469

А) неверное равенство;

В) верное неравенство;

С) неверное неравенство;

D) верное равенство.


  1. Найди запись, которая может заменить выражение 16*440 так, чтобы значение не изменилось.

А) 16*(400*40)

В) 4*4 +11 *40

С) 16*(11*40)

D) 4*(4+11)*40


  1. Выбери верное равенство:

А) 63 : 9 + 18 :2 = (63 + 18) : 9

В) (63 + 18) : 9 = 63 : 9 + 18 : 9

С) (63 + 18) : 9 = 63 : 9 +18

D) (63 + 18) : = 63 + 18 : 9


  1. Найди выражение, значение которого не изменяется, если убрать скобки:

А) (70+90)*8 В) (70+90) : 8 С) 70 * (90*8) D) 60*(80+7)

  1. Укажи верное неравенство.

А) 7т 030кг > 7т 3ц

В) 7т 030кг > 7т30кг

C) 7т 030кг

D) 7т 030кг


  1. Найди значение, которое получится при делении 54км на 6м.

А) 9 В) 9км С) 9000 D) 900м

8. Какая пара наименований единиц величин сделает равенство 2________ = 200 _______ верным,

если наименование брать в том порядке, как они записаны?

А) кг и г В) м и см С) ч и мин D) дм и см


  1. Каким будет значение выражения: а : 1 + а * 0

А) в В) 0 С) 1 D) а


  1. Укажи первое неполное делимое в действии 13578 : 6

А) 13 В) 1 С) 135 D) 1357


  1. Укажи значение неизвестного в уравнении:

8 * х = 8 + 8 + 8

А) 2 В)3 С) 4 D) 15


  1. В каком уравнении можно найти значение Х?

А) 10 : х = 5000

В) 5 * х = 4055

С) 1089 * х = 560

D) х + 35698 = 987


  1. 81 – это квадрат числа…

А) 9 В) 3 С) 90 D) 30

  1. Закончи правило: чтобы найти время движения, нужно длину пройденного пути … на скорость движения.

А) увеличить В) разделить C)уменьшить D) умножить

  1. Мотоциклист проехал 120км за 2 часа. За сколько часов он проедет 240 при той же скорости?

А) за 3 часа В) за 4 часа С) за 5 часов D) за 1 час


  1. На катке было 30 взрослых, что в 6 раз меньше, чем детей. Сколько детей было на катке?

А) 5 чел. В) 180 чел. С) 36 чел. D) 24 чел.

  1. 12 карандашей стоят 240 тенге. Сколько стоит 1 карандаш?

А) 10 тенге В) 20 тенге С) 100 тенге D) 12 тенге

  1. Площади двух прямоугольников равны. Длина первого прямоугольника 8см., а ширина 5см., Чему равна ширина второго прямоугольника, если его длина равна 10см.

А) 40см В) 4см С) 3см D) 130см


  1. Длина и ширина вместе составляет 30см. Какова длина и ширина, если длина в 5 раз больше?

А) 24 и 6 В) 25 и 5 С) 15 и 15 D) 12 и 18


  1. Укажи длину ломаной, состоящей из 8 равных звеньев, если длина такого звена равна 3см.

А) 11см В) 24см С) 27см D) 30см

Лист для самостоятельной работы на уроке математики по теме «Неравенства» (3 класс)

Фамилия, имя____________________________________________________ Лист для самостоятельной работы       Дата ___________________ 30: 3=         80: 20=         37*2=         3*28=           39: 3=          48: 2=       70: 35= 77: 11=       20*4=           60: 30=       48: 2=           2*29=           90: 5=      75: 3=  1. Найдите неравенства: а) а + в = с               в) с > а        б) а + в > c              г)  а + в = с + а 2. Поставь знак действия, чтобы получилось верное неравенство.   а) 9 5 < 50                           б) 0 20 < 20 3. Подбери и вставь числа, чтобы получились верные неравенства.  а) 18 + 20 <  * 2       (30, 20,15)     б) 5 * 3 > 5 *             (4, 2, 5)       4.Найди верное неравенство. а) 5 * 3 < 10 б) 64 : 8 > 5 * 7            в) 48 : 6 < 10     5.Решить систему   х  x  Где в жизни я могу использовать неравенства?       (есть, нет)   6 8 Фамилия, имя____________________________________________________ Лист для самостоятельной работы       Дата ___________________ 30: 3=         80: 20=         37*2=         3*28=           39: 3=          48: 2=       70: 35= 77: 11=       20*4=           60: 30=       48: 2=           2*29=           90: 5=      75: 3=  1. Найдите неравенства: а) а + в = с               в) с > а        б) а + в > c              г)  а + в = с + а 2. Поставь знак действия, чтобы получилось верное неравенство.   а) 9 5 < 50                           б) 0 20 < 20 3. Подбери и вставь числа, чтобы получились верные неравенства.  а) 18 + 20 <  * 2       (30, 20,15)     б) 5 * 3 > 5 *             (4, 2, 5)       4.Найди верное неравенство. а) 5 * 3 < 10 б) 64 : 8 > 5 * 7            в) 48 : 6 < 10     5.Решить систему   6 8       (есть, нет)  х  x  Где в жизни я могу использовать неравенства? Разминка – тест.  Проверим, как справляемся с неравенствами. 6 8   верно (б, в)       (есть, нет)        верно: нет, т.к. нет одинаковых чисел при решении неравенства 1. Найдите неравенства: а) а + в = с               в) с > а  б) а + в > c              г)  а + в = с + а  2. Поставь знак, чтобы получилось верное неравенство. а) 9 5 < 50            ( +, ­, х, 🙂                 верно (+, ­, х) б) 0 20 <               ( +, ­, х, : )                верно (х, 🙂 3. Подбери и вставь числа, чтобы получились верные неравенства. а) 18 + 20 <  * 2           (20, 15, 30)       прав. (20, 30) б) 5 * 3 > 5 *                ( 4, 2, 5)            прав. (2) 4.Найди верное неравенство. а) 5 * 3 < 10 б) 64 : 8 > 5 * 7           верно  (в) в) 48 : 6 < 10     5.Решить систему   х  x  Работа в тетрадях. 1. Решить неравенства  путем подбора. а) х < 1, (если х = 0)     а < 0, (не имеет решения)     у > 0, (если у – любые числа например, 1 > 0, 5 >0)     x : х < 2 (любое число, кроме нуля, т.к. при  делении число на это число дает 1)      5 : 5 < 2          1 < 2     100 : 100 < 2               1 < 2 б) у + у < 5  ­ путем подбора. ( в правой части большее число, чем в левой. В левой будет   меньше 5, это 4,3,2,1,0)      у = 2, 1    Проверяю значение суммы     2 + 2 < 5     1 + 1 < 5    Я беру числа 2, 1, 0    Почему их?    Потому что у нас сумма одинаковых  слагаемых. в) R : 7 < 6                       R = 35, 28, 21, 14, 7, 0                                          35: 7  < 6                                          5 < 6 г) Решить неравенство с помощью уравнения.         5 * х > 10                     5 * x = 10         x > 10 : 5                      х = 10 : 5         х > 2                             x = 2     5 * 3 > 10                      5 * 2 = 10   15 > 10                          10 = 10

Решение вопросов со словами о неравенстве

(Вы можете сначала прочитать Введение в неравенство и решение неравенств.)

В алгебре у нас есть вопросы о неравенстве, например:

Сэм и Алекс играют в одной футбольной команде.


В минувшую субботу Алекс забил на 3 гола больше, чем Сэм, но вместе они забили меньше 9 голов.

Какое возможное количество голов забил Алекс?

Как мы их решаем?

Уловка состоит в том, чтобы разбить решение на две части:

Превратите английский в алгебру.

Затем решите с помощью алгебры.

Как английский язык превращается в алгебру

Превратить английский в алгебру помогает:

  • Прочтите сначала все
  • Сделайте набросок, если нужно
  • Присвойте буквы значениям
  • Найдите или обработайте формулы

Мы также должны записать , что на самом деле требуется для , чтобы мы знали, куда мы идем и когда мы приедем!

Лучший способ узнать это — на примере, поэтому давайте попробуем наш первый пример:

Сэм и Алекс играют в одной футбольной команде.


В минувшую субботу Алекс забил на 3 гола больше, чем Сэм, но вместе они забили меньше 9 голов.

Какое возможное количество голов забил Алекс?

Письма о назначении:

  • количество голов, забитых Алексом: A
  • количество голов, забитых Сэмом: S

Мы знаем, что Алекс забил на 3 гола больше, чем Сэм, поэтому: A = S + 3

А мы знаем, что вместе они забили меньше 9 голов: S + A <9

Нас спрашивают, сколько голов мог бы забить Алекс: A

Решить:

Начать с: S + A <9

A = S + 3, поэтому: S + (S + 3) <9

Упростить: 2S + 3 <9

Вычтем 3 с обеих сторон: 2S <9 - 3

Упростить: 2S <6

Разделите обе стороны на 2: S <3

Сэм забил менее 3 голов, что означает, что Сэм мог забить 0, 1 или 2 гола.

Алекс забил на 3 гола больше, чем Сэм, поэтому Алекс мог забить 3, 4 или 5 голов .

Чек:

  • Когда S = 0, тогда A = 3 и S + A = 3, и 3 <9 правильно
  • Когда S = 1, тогда A = 4 и S + A = 5, и 5 <9 правильно
  • Когда S = 2, тогда A = 5 и S + A = 7, и 7 <9 правильно
  • (Но когда S = 3, тогда A = 6 и S + A = 9, а 9 <9 неверно)

Еще много примеров!

Пример: Из 8 щенков девочек больше, чем мальчиков.

Сколько может быть девочек-щенков?

Письма о назначении:

  • Количество девочек: г
  • количество мальчиков: б

Мы знаем, что есть 8 щенков, поэтому: g + b = 8, что может быть преобразовано в

б = 8 — г

Мы также знаем, что девочек больше, чем мальчиков, поэтому:

г> б

У нас спрашивают количество щенков: г

Решить:

Начать с: g> b

b = 8 — g , поэтому: g> 8 — g

Добавьте g к обеим сторонам: g + g> 8

Упростить: 2g> 8

Разделите обе стороны на 2: г> 4

Итак, девочек может быть 5, 6, 7 или 8.

А может быть 8 девочек? Тогда бы вообще не было мальчиков, и вопрос по этому поводу не ясен (иногда вопросы такие).

Чек

  • Когда g = 8, тогда b = 0 и g> b правильно (но разрешено ли b = 0?)
  • Когда g = 7, тогда b = 1 и правильное g> b
  • Когда g = 6, тогда b = 2 и g> b правильно
  • Когда g = 5, тогда b = 3 и g> b правильно
  • (Но если g = 4, то b = 4 и g> b неверно)

Быстрый пример:

Пример: Джо участвует в гонке, где ему нужно ехать на велосипеде и бегать.

Он проезжает на велосипеде дистанцию ​​25 км, а затем пробегает 20 км. Его средняя скорость бега составляет половину его средней скорости езды на велосипеде.

Джо завершает гонку менее чем за 2,5 часа, что мы можем сказать о его средней скорости?

Письма о назначении:

  • Средняя скорость движения: с
  • Итак, средняя скорость езды на велосипеде: 2 с

Формулы:

  • Скорость = Расстояние Время
  • Что можно изменить на: Время = Расстояние Скорость

Нас спрашивают о его средних скоростях: с и 2 с

Гонка делится на две части:

1.Велоспорт
  • Расстояние = 25 км
  • Средняя скорость = 2 с км / ч
  • Итак Время = Расстояние Средняя скорость = 25 2 с часов
2. Работает
  • Расстояние = 20 км
  • Средняя скорость = с км / ч
  • Итак Время = Расстояние Средняя скорость = 20 с часов

Джо завершает гонку менее чем за 2,5 часа

  • Общее время <2½
  • 25 2s + 20 s <2½

Решить:

Начать с: 25 2s + 20 s <2½

Умножить все члены на 2 с: 25 + 40 <5 с

Упростить: 65 <5 с

Разделите обе стороны на 5: 13

Поменять местами стороны: с> 13

Значит, его средняя скорость бега больше 13 км / ч, а его средняя скорость езды на велосипеде больше 26 км / ч

В этом примере мы можем использовать сразу два неравенства:

Пример: скорость

v м / с шара, брошенного прямо в воздух, определяется как v = 20 — 10t , где t — время в секундах.

В какое время скорость будет от 10 до 15 м / с?

Письма:

  • скорость в м / с: v
  • время в секундах: t

Формула:

У нас спрашивают время t , когда v находится между 5 и 15 м / с:

10

10 <20 - 10 т <15

Решить:

Начать с: 10 <20 - 10т <15

Вычтем 20 из каждого: 10-20 <20-10t - 20 <15-20

Упростить: −10 <−10t <−5

Разделим каждое на 10: −1 <−t <−0.5

Изменить знаки и отменить неравенства: 1> t> 0,5

Лучше сначала показать меньшее число
, поэтому поменяйте местами: 0,5

Таким образом, скорость составляет от 10 до 15 м / с между 0,5 и 1 секундой позже.

И достаточно жесткий пример для завершения:

Пример: прямоугольная комната вмещает не менее 7 столов, каждый из которых имеет площадь 1 квадратный метр.Периметр комнаты 16 м.


Какой может быть ширина и длина комнаты?

Сделайте набросок: мы не знаем размеров столов, только их площадь, могут они идеально поместиться или нет!

Письма о назначении:

  • Длина помещения: L
  • Ширина помещения: Вт

Формула для периметра: 2 (Ш + Д) , и мы знаем, что это 16 м

  • 2 (Ш + Д) = 16
  • Вт + Д = 8
  • Д = 8 — Ш

Мы также знаем, что площадь прямоугольника равна ширине, умноженной на длину: Площадь = Ш × Д

И площадь должна быть больше или равна 7:

Нас спрашивают о возможных значениях W и L

Решим:

Начать с: Ш × Д ≥ 7

Заменитель L = 8 — W: W × (8 — W) ≥ 7

Expand: 8W — W 2 ≥ 7

Переместите все термины в левую часть: W 2 — 8W + 7 ≤ 0

Это квадратное неравенство.Ее можно решить разными способами, здесь мы решим ее, заполнив квадрат:

Переместите числовой член 7 в правую часть неравенства: W 2 — 8W ≤ −7

Заполните квадрат в левой части неравенства и уравновесите его, прибавив такое же значение к правая часть неравенства: W 2 — 8W + 16 ≤ −7 + 16

Упростить: (W — 4) 2 ≤ 9

Извлеките квадратный корень из обеих частей неравенства: −3 ≤ W — 4 ≤ 3

Да, у нас есть два неравенства, потому что 3 2 = 9 И (−3) 2 = 9

Добавьте 4 к обеим сторонам каждого неравенства: 1 ≤ W ≤ 7

Таким образом, ширина должна быть между 1 м и 7 м (включительно), а длина — 8 — ширина .

Чек:

  • Скажем, W = 1, тогда L = 8−1 = 7 и A = 1 x 7 = 7 м 2 (подходит ровно для 7 таблиц)
  • Скажем, W = 0,9 (меньше 1), тогда L = 7,1 и A = 0,9 x 7,1 = 6,39 м 2 (7 не подходят)
  • Скажем, W = 1,1 (чуть выше 1), тогда L = 6,9 и A = 1,1 x 6,9 = 7,59 м 2 (7 легко помещаются)
  • Аналогично для W около 7 м

Решайте неравенства с помощью программы «Пошаговое решение математических задач»

В главе 2 мы установили правила решения уравнений с использованием чисел арифметики.Теперь, когда мы изучили операции с числами со знаком, мы будем использовать те же правила для решения уравнений, содержащих отрицательные числа. Мы также изучим методы решения и построения графиков неравенств с одним неизвестным.

РЕШЕНИЕ УРАВНЕНИЙ НА ЗАПИСАННЫХ ЧИСЛАХ

ЗАДАЧИ

По завершении этого раздела вы сможете решать уравнения, содержащие числа со знаком.

Пример 1 Решите относительно x и проверьте: x + 5 = 3

Решение

Используя те же процедуры, что и в главе 2, мы вычитаем 5 из каждой части уравнения, получая

Пример 2 Решите относительно x и проверьте: — 3x = 12

Решение

Разделив каждую сторону на -3, получаем

Всегда проверяйте исходное уравнение.

Другой способ решения уравнения
3x — 4 = 7x + 8
— сначала вычесть 3x из обеих сторон, получив
-4 = 4x + 8,
, затем вычесть 8 с обеих сторон и получить
-12 = 4x .
Теперь разделите обе стороны на 4, получив
— 3 = x или x = — 3.

Сначала удалите круглые скобки. Затем следуйте процедуре, описанной в главе 2.

ЛИТЕРАЛЬНЫЕ УРАВНЕНИЯ

ЗАДАЧИ

По завершении этого раздела вы сможете:

  1. Определите буквальное уравнение.
  2. Примените ранее изученные правила для решения буквальных уравнений.

Уравнение, состоящее из нескольких букв, иногда называют буквальным уравнением . Иногда бывает необходимо решить такое уравнение для одной из букв через другие. Пошаговая процедура, описанная и использованная в главе 2, остается действительной после удаления любых символов группировки.

Пример 1 Решите относительно c: 3 (x + c) — 4y = 2x — 5c

Решение

Сначала удалите круглые скобки.

Здесь мы отмечаем, что, поскольку мы решаем для c, мы хотим получить c с одной стороны и все другие члены с другой стороны уравнения. Таким образом, получаем

Помните, abx — это то же самое, что 1abx.
Делим на коэффициент при x, который в данном случае равен ab.

Решите уравнение 2x + 2y — 9x + 9a, сначала вычтя 2.v из обеих частей. Сравните полученное решение с полученным в примере.

Иногда форму ответа можно изменить. В этом примере мы могли бы умножить числитель и знаменатель ответа на (- l) (это не меняет значения ответа) и получить

Преимущество этого последнего выражения перед первым в том, что в ответе не так много отрицательных знаков.

Умножение числителя и знаменателя дроби на одно и то же число — это использование фундаментального принципа дробей.

Наиболее часто используемые буквальные выражения — это формулы из геометрии, физики, бизнеса, электроники и т. Д.

Пример 4 — это формула площади трапеции. Решите для c.

Трапеция имеет две параллельные стороны и две непараллельные стороны. Параллельные стороны называются основаниями.
Удаление скобок не означает их простое стирание. Мы должны умножить каждый член в круглых скобках на множитель, стоящий перед скобками.
Менять форму ответа не обязательно, но вы должны уметь распознать правильный ответ, даже если форма не та.

Пример 5 — это формула, дающая проценты (I), полученные за период D дней, когда известны основная сумма (p) и годовая ставка (r). Найдите годовую ставку, когда известны сумма процентов, основная сумма и количество дней.

Решение

Задача требует решения для р.

Обратите внимание, что в этом примере r оставлено справа, и поэтому вычисление было проще. При желании мы можем переписать ответ по-другому.

ГРАФИЧЕСКИЕ НЕРАВЕНСТВА

ЗАДАЧИ

По завершении этого раздела вы сможете:

  1. Используйте символ неравенства для обозначения относительного положения двух чисел на числовой прямой.
  2. График неравенств на числовой прямой.

Мы уже обсуждали набор рациональных чисел как числа, которые могут быть выражены как отношение двух целых чисел.Существует также набор чисел, называемых иррациональными числами , , которые нельзя выразить как отношение целых чисел. В этот набор входят такие числа как и так далее. Набор, состоящий из рациональных и иррациональных чисел, называется действительными числами.

Учитывая любые два действительных числа a и b, всегда можно заявить, что Часто нас интересует только то, равны ли два числа или нет, но бывают ситуации, когда мы также хотим представить относительный размер чисел, которые не равны. равный.

Символы являются символами неравенства или отношениями порядка и используются для отображения относительных размеров значений двух чисел. Обычно мы читаем этот символ как «больше чем». Например, a> b читается как «a больше, чем b». Обратите внимание, что мы заявили, что обычно читаем

а


Какое положительное число можно добавить к 2, чтобы получить 5?


Проще говоря, это определение утверждает, что a меньше b, если мы должны что-то добавить к a, чтобы получить b.Конечно, «что-то» должно быть положительным.

Если вы думаете о числовой прямой, вы знаете, что добавление положительного числа равносильно перемещению вправо по числовой прямой. Это приводит к следующему альтернативному определению, которое может быть легче визуализировать.

Пример 1 3


Мы также можем написать 6> 3.

Пример 2 — 4


Мы также можем написать 0> — 4.

Пример 3 4> — 2, потому что 4 находится справа от -2 в числовой строке.


Пример 4 — 6


Математическое утверждение x

Вы понимаете, почему невозможно найти наибольшее число меньше трех?

На самом деле назвать число x, которое является наибольшим числом меньше 3, — задача невыполнимая. Однако это может быть указано в числовой строке.Для этого нам нужен символ, обозначающий значение такого оператора, как x

Символы (и), используемые в числовой строке, указывают на то, что конечная точка не включена в набор.

Пример 5 График x

Решение


Обратите внимание, что на графике есть стрелка, указывающая на то, что линия продолжается без конца влево.

На этом графике представлено каждое действительное число меньше 3.

Пример 6 График x> 4 на числовой прямой.

Решение


На этом графике представлено каждое действительное число больше 4.

Пример 7 График x> -5 на числовой прямой.

Решение


На этом графике представлены все действительные числа больше -5.

Пример 8 Постройте числовой график, показывающий, что x> — 1 и x

Решение


Выписка x> — 1 и x

На этом графике представлены все действительные числа от -1 до 5.

Пример 9 График — 3

Решение

Если мы хотим включить конечную точку в набор, мы используем другой символ:. Мы читаем эти символы как «равно или меньше» и «равно или больше».

Пример 10 x>; 4 обозначает число 4 и все действительные числа справа от 4 в числовой строке.

Символы [и] в числовой строке указывают, что конечная точка включена в набор.

Вы обнаружите, что такое использование круглых и квадратных скобок согласуется с их использованием в будущих курсах математики.

На этом графике представлено число 1 и все действительные числа больше 1.

На этом графике представлено число 1 и все действительные числа, меньшие или равные — 3.

Пример 13 Напишите алгебраическое утверждение, представленное следующим графиком.

Пример 14 Напишите алгебраическое выражение для следующего графика.

На этом графике представлены все действительные числа от -4 до 5 , включая -4 и 5.

Пример 15 Напишите алгебраическое выражение для следующего графика.

Этот график включает 4, но , а не -2.

Пример 16 График на числовой прямой.

Решение

В этом примере возникает небольшая проблема. Как мы можем указать на числовой строке? Если мы оценим суть дела, то другой человек может неправильно истолковать это утверждение. Не могли бы вы сказать, представляет ли эта точка или может быть? Поскольку цель графика — пояснить, всегда обозначает конечную точку.

График используется для передачи утверждения. Вы всегда должны называть нулевую точку, чтобы показать направление, а также конечную точку или точки, если быть точным.

УСТРАНЕНИЕ НЕРАВЕНСТВ

ЗАДАЧИ

По завершении этого раздела вы сможете решать неравенства с одним неизвестным.

Решения неравенств обычно основаны на тех же основных правилах, что и уравнения. Есть одно исключение, которое мы скоро обнаружим. Однако первое правило аналогично тому, что используется при решении уравнений.

Если одно и то же количество добавляется к каждой стороне неравенства , результаты будут неравными в том же порядке.

Пример 1 Если 5

Пример 2 Если 7

Мы можем использовать это правило для решения некоторых неравенств.

Пример 3 Решить относительно x: x + 6

Решение

Если мы прибавим -6 к каждой стороне, мы получим

Изобразив это решение на числовой прямой, получим

Обратите внимание, что процедура такая же, как и при решении уравнений.

Теперь мы воспользуемся правилом сложения, чтобы проиллюстрировать важную концепцию, касающуюся умножения или деления неравенств.

Предположим, что x> a.

Теперь добавьте — x к обеим сторонам по правилу сложения.

Помните, добавление одинаковой величины к обеим сторонам неравенства не меняет его направления.

Теперь добавьте -a с обеих сторон.

Последний оператор — a> -x можно переписать как — x <-a. Поэтому мы можем сказать: «Если x> a, то — x

Если неравенство умножается или делится на отрицательное число , результаты будут неравными в порядке , противоположном .

Например: Если 5> 3, то -5

Пример 5 Решите относительно x и изобразите решение: -2x> 6

Решение

Чтобы получить x в левой части, мы должны разделить каждый член на — 2. Обратите внимание, что, поскольку мы делим на отрицательное число, мы должны изменить направление неравенства.

Обратите внимание, что как только мы делим на отрицательную величину, мы должны изменить направление неравенства.

Обратите внимание на этот факт. Каждый раз, когда вы делите или умножаете на отрицательное число, вы должны изменять направление символа неравенства. Это единственное различие между решением уравнений и решением неравенств.

Когда мы умножаем или делим на положительное число, изменений нет. Когда мы умножаем или делим на отрицательное число, направление неравенства меняется. Будьте осторожны — это источник многих ошибок.

После того, как мы удалили круглые скобки и остались только отдельные члены в выражении, процедура поиска решения почти такая же, как в главе 2.

Давайте теперь рассмотрим пошаговый метод из главы 2 и отметим разницу при решении неравенств.

Первые Исключите дроби, умножив все члены на наименьший общий знаменатель всех дробей. (Без изменений, когда мы умножаем на положительное число.)
Второй Упростите, комбинируя одинаковые члены с каждой стороны неравенства. (Без изменений)
Третий Сложите или вычтите количества, чтобы получить неизвестное с одной стороны и числа с другой.(Без изменений)
Четвертый Разделите каждый член неравенства на коэффициент неизвестной. Если коэффициент положительный, неравенство останется прежним. Если коэффициент отрицательный, неравенство будет отменено. (Это важное различие между уравнениями и неравенствами.)

Единственное возможное отличие — на последнем этапе.

Что нужно делать при делении на отрицательное число?

Не забудьте пометить конечную точку.

РЕЗЮМЕ

Ключевые слова

  • Литеральное уравнение — это уравнение, состоящее из более чем одной буквы.
  • Символы — это символа неравенства или отношения порядка .
  • a a находится слева от b в строке действительного числа.
  • Двойные символы: указывают, что конечные точки включены в набор решений .

Процедуры

  • Чтобы решить буквальное уравнение для одной буквы через другие, выполните те же действия, что и в главе 2.
  • Чтобы решить неравенство, используйте следующие шаги:
    Шаг 1 Исключите дроби, умножив все члены на наименьший общий знаменатель всех дробей.
    Шаг 2 Упростите, объединив одинаковые термины с каждой стороны неравенства.
    Шаг 3 Сложите или вычтите величины, чтобы получить неизвестное с одной стороны и числа с другой.
    Шаг 4 Разделите каждый член неравенства на коэффициент неизвестной. Если коэффициент положительный, неравенство останется прежним.Если коэффициент отрицательный, неравенство будет отменено.
    Шаг 5 Проверьте свой ответ.

Письменные неравенства — Алгебра 1

Если вы считаете, что контент, доступный через Веб-сайт (как определено в наших Условиях обслуживания), нарушает или другие ваши авторские права, сообщите нам, отправив письменное уведомление («Уведомление о нарушении»), содержащее в информацию, описанную ниже, назначенному ниже агенту.Если репетиторы университета предпримут действия в ответ на ан Уведомление о нарушении, оно предпримет добросовестную попытку связаться со стороной, которая предоставила такой контент средствами самого последнего адреса электронной почты, если таковой имеется, предоставленного такой стороной Varsity Tutors.

Ваше Уведомление о нарушении прав может быть отправлено стороне, предоставившей доступ к контенту, или третьим лицам, таким как в качестве ChillingEffects.org.

Обратите внимание, что вы будете нести ответственность за ущерб (включая расходы и гонорары адвокатам), если вы существенно искажать информацию о том, что продукт или действие нарушает ваши авторские права.Таким образом, если вы не уверены, что контент находится на Веб-сайте или по ссылке с него нарушает ваши авторские права, вам следует сначала обратиться к юристу.

Чтобы отправить уведомление, выполните следующие действия:

Вы должны включить следующее:

Физическая или электронная подпись правообладателя или лица, уполномоченного действовать от их имени; Идентификация авторских прав, которые, как утверждается, были нарушены; Описание характера и точного местонахождения контента, который, по вашему мнению, нарушает ваши авторские права, в \ достаточно подробностей, чтобы позволить репетиторам университетских школ найти и точно идентифицировать этот контент; например нам требуется а ссылка на конкретный вопрос (а не только на название вопроса), который содержит содержание и описание к какой конкретной части вопроса — изображению, ссылке, тексту и т. д. — относится ваша жалоба; Ваше имя, адрес, номер телефона и адрес электронной почты; а также Ваше заявление: (а) вы добросовестно считаете, что использование контента, который, по вашему мнению, нарушает ваши авторские права не разрешены законом, владельцем авторских прав или его агентом; (б) что все информация, содержащаяся в вашем Уведомлении о нарушении, является точной, и (c) под страхом наказания за лжесвидетельство, что вы либо владелец авторских прав, либо лицо, уполномоченное действовать от их имени.

Отправьте жалобу нашему уполномоченному агенту по адресу:

Чарльз Кон Varsity Tutors LLC
101 S. Hanley Rd, Suite 300
St. Louis, MO 63105

Или заполните форму ниже:

Как найти решение неравенства с умножением

Если вы считаете, что контент, доступный через Веб-сайт (как определено в наших Условиях обслуживания), нарушает или другие ваши авторские права, сообщите нам, отправив письменное уведомление («Уведомление о нарушении»), содержащее в информацию, описанную ниже, назначенному ниже агенту.Если репетиторы университета предпримут действия в ответ на ан Уведомление о нарушении, оно предпримет добросовестную попытку связаться со стороной, которая предоставила такой контент средствами самого последнего адреса электронной почты, если таковой имеется, предоставленного такой стороной Varsity Tutors.

Ваше Уведомление о нарушении прав может быть отправлено стороне, предоставившей доступ к контенту, или третьим лицам, таким как в качестве ChillingEffects.org.

Обратите внимание, что вы будете нести ответственность за ущерб (включая расходы и гонорары адвокатам), если вы существенно искажать информацию о том, что продукт или действие нарушает ваши авторские права.Таким образом, если вы не уверены, что контент находится на Веб-сайте или по ссылке с него нарушает ваши авторские права, вам следует сначала обратиться к юристу.

Чтобы отправить уведомление, выполните следующие действия:

Вы должны включить следующее:

Физическая или электронная подпись правообладателя или лица, уполномоченного действовать от их имени; Идентификация авторских прав, которые, как утверждается, были нарушены; Описание характера и точного местонахождения контента, который, по вашему мнению, нарушает ваши авторские права, в \ достаточно подробностей, чтобы позволить репетиторам университетских школ найти и точно идентифицировать этот контент; например нам требуется а ссылка на конкретный вопрос (а не только на название вопроса), который содержит содержание и описание к какой конкретной части вопроса — изображению, ссылке, тексту и т. д. — относится ваша жалоба; Ваше имя, адрес, номер телефона и адрес электронной почты; а также Ваше заявление: (а) вы добросовестно считаете, что использование контента, который, по вашему мнению, нарушает ваши авторские права не разрешены законом, владельцем авторских прав или его агентом; (б) что все информация, содержащаяся в вашем Уведомлении о нарушении, является точной, и (c) под страхом наказания за лжесвидетельство, что вы либо владелец авторских прав, либо лицо, уполномоченное действовать от их имени.

Отправьте жалобу нашему уполномоченному агенту по адресу:

Чарльз Кон Varsity Tutors LLC
101 S. Hanley Rd, Suite 300
St. Louis, MO 63105

Или заполните форму ниже:

Неравенство абсолютных значений

Неравенство абсолютных значений Абсолютное неравенство

Вот шаги, которые необходимо выполнить при решении абсолютного значения неравенства:

  1. Выделите выражение абсолютного значения в левой части неравенство.
  2. Если число с другой стороны знака неравенства равно отрицательный, ваше уравнение либо не имеет решения, либо все действительные числа являются решениями. Используйте знак каждой стороны неравенства, чтобы решить, какой из этих случаев держит. Если число по другую сторону знака неравенства положительное, перейти к шагу 3.
  3. Удалите полосы абсолютных значений, установив составное неравенство. Тип знака неравенства в задаче подскажет нам, как настроить сложное неравенство.
  4. Если ваша проблема имеет знак больше, чем (ваша проблема теперь говорит, что абсолютное значение больше числа), затем установите сложное неравенство «или», которое выглядит следующим образом:

    (количество внутри абсолютного значения) <- (число на другом боковая сторона)
    ИЛИ
    (количество внутри абсолютного значения)> (число на другой стороне)

    Такая же установка используется для ³ подписать.

    Если ваше абсолютное значение на меньше числа, тогда установить составное неравенство из трех частей, которое выглядит так:

    — (число на другой стороне) <(количество внутри абсолютного значение) <(число на другой стороне)

    Такая же установка используется для £ знак

  5. Решите неравенства.


Поначалу этот процесс может немного сбивать с толку, так что будьте пациент, пока учится решать эти задачи. Давайте посмотрим на несколько примеров.

Пример 1: | x + 4 | — 6 <9

Шаг 1: Изолировать абсолют значение | х + 4 | — 6 <9

| x + 4 | <15

Шаг 2: Номер включен другая сторона отрицательная? Нет, это положительное число, 15.Переходим к шагу 3.
Шаг 3: Установить сложное неравенство Знак неравенства в нашей задаче меньше знак, поэтому мы установим неравенство из 3 частей:

-15 <х + 4 <15

Шаг 4: Решить составное неравенство -19 <х <11

Пример 2: | 2x 1 | — 7 ³ -3

Шаг 1: Изолировать абсолют значение | 2x 1 | — 7 ³ -3

| 2x 1 | ³ 4

Шаг 2: Is число на другой стороне отрицательное число? Нет, это положительное число, 4.Переходим к шагу 3.
Шаг 3: Установить сложное неравенство Знак неравенства в нашей задаче больше или знак равенства, поэтому составим составное неравенство со словом «или»:

2x 1 £ -4 или 2x 1 ³ 4

Шаг 4: Решить неравенства 2x 1 £ -4 или 2x 1 ³ 4

2x £ -3 или 2x ³ 5

x £ -3/2 или x ³ 5/2


Пример 3: | 5x + 6 | + 4 <1

Шаг 1: Изолировать абсолютное значение | 5x + 6 | + 4 <1

| 5x + 6 | <-3

Шаг 2: Is число на другой стороне отрицательное число? Да, это отрицательное число, -3.

Посмотрите на признаки каждой стороны неравенства для определения решения проблемы:

| 5x + 6 | <-3

положительный <отрицательный

Это утверждение неверно, поэтому решения нет. к этой проблеме.


Пример 4: | 3x 4 | + 9> 5

Шаг 1: Изолировать абсолютное значение | 3х 4 | + 9> 5

| 3x 4 | > -4

Шаг 2: Is число на другой стороне отрицательное число? Да, это отрицательное число, -4.

Посмотрите на признаки каждой стороны неравенства для определения решения проблемы:

| 3x 4 | > -4

положительный> отрицательный

Это утверждение всегда верно, поэтому решение проблема — все действительные числа

единиц чтения: устранение неравенств в один шаг | Конечная математика

Неравенства похожи на уравнения в том, что они показывают взаимосвязь между двумя выражениями.Мы решаем и графически изображаем неравенства аналогично уравнениям. Однако есть некоторые отличия, о которых мы поговорим в этой главе. Основное отличие состоит в том, что для линейных неравенств ответом является интервал значений, тогда как для линейного уравнения ответом чаще всего является одно значение.

При написании неравенств мы используем следующие символы

> больше

[латекс] \ ge [/ latex] больше или равно

<меньше

[латекс] \ le [/ latex] меньше или равно

Запишите и изобразите неравенства в одной переменной на числовой прямой

Начнем с простого неравенства x> 3

Мы читаем это неравенство как «x больше 3.«Решение — это набор всех действительных чисел, которые больше трех. Мы часто представляем множество решений неравенства числовой линией.

Рассмотрим еще одно простое неравенство [латекс] x \ le4 [/ latex]

Мы читаем это неравенство как «x меньше или равно 4». Решение — это набор всех действительных чисел, которые равны четырем или меньше четырех. Построим это множество решений на числовой прямой.

В графе мы используем пустой кружок для конечной точки строгого неравенства (x> 3) и закрашенный кружок, если включен знак равенства (x [latex] \ le [/ latex] 4).

Пример 1

На числовой прямой нанесите следующие неравенства.

  1. х <−3
  2. [латекс] x \ ge6 [/ латекс]
  3. x> 0
  4. [латекс] x \ le8 [/ латекс]

Решения

  1. Неравенство x <−3 представляет все действительные числа, которые меньше −3. Число −3 не входит в решение и обозначено белым кружком на графике.
  2. Неравенство [латекс] x \ ge6 [/ latex] представляет все действительные числа, которые больше или равны шести.Число шесть включено в решение и обозначено на графике замкнутым кружком.
  3. Неравенство x > 0 представляет все действительные числа, которые больше нуля. Число ноль не входит в решение и обозначено белым кружком на графике.
  4. Неравенство [латекс] x \ le8 [/ latex] представляет все действительные числа, которые меньше или равны восьми. Число восемь включено в решение и обозначено на графике замкнутым кружком.

Пример 2

Напишите неравенство, которое представлено на каждом графике.

Решения

  1. x <−12
  2. x > 540
  3. x <65
  4. [латекс] x \ ge83 [/ латекс]

В реальной жизни неравенство проявляется повсюду. Вот несколько простых примеров реальных приложений.

Пример 3

Запишите каждое утверждение в виде неравенства и обозначьте его числовой прямой.

  1. Чтобы получить бесплатную проверку, на вашем текущем счете необходимо поддерживать баланс не менее $ 2500.
  2. Чтобы кататься на американских горках «Thunderbolt», вы должны быть ростом не менее 48 дюймов.
  3. Чтобы получить бесплатный вход в зоопарк Сан-Диего, вам должно быть меньше 3 лет.
  4. Ограничение скорости на межштатной автомагистрали составляет 65 миль в час.

Решения

  1. Неравенство записывается как [latex] x \ ge2500 [/ latex]. Слова «как минимум» означают, что в набор решений включена стоимость 2500 долларов.
  2. Неравенство записывается как [latex] x \ ge48 [/ latex]. Слова «по крайней мере» означают, что значение 48 дюймов включено в набор решений.
  3. Неравенство записывается как x <3.
  4. Ограничение скорости означает наивысшую допустимую скорость, поэтому неравенство записывается как [latex] x \ le65 [/ latex].

Чтобы решить неравенство, мы должны изолировать переменную с одной стороны от знака неравенства. Чтобы изолировать переменную, мы используем те же базовые методы, которые используются при решении уравнений.Для неравенств этого типа:

x a < b или x a > b

Мы изолируем x, добавив константу a к обеим сторонам неравенства.

Пример 4

Решите каждое неравенство и изобразите набор решений.

  1. х − 3 <10
  2. х − 1> −10
  3. [латекс] x − 1 \ le − 5 [/ латекс]
  4. [латекс] x − 20 \ ge14 [/ латекс]

Решения

  1. Чтобы решить неравенство x- 3 <10 Упростите:
    x <13

  2. Решить неравенство x — 1> -10
  3. Решить неравенство [латекс] x-1 \ le-5 [/ latex].Упростить:
    [латекс] x \ le-4 [/ латекс]

  4. Решить неравенство [латекс] x-20 \ le14 [/ latex]

Для неравенств этого типа:

x + 1 b
Мы выделяем x, вычитая константу a с обеих сторон неравенства.

Пример 5

Решите каждое неравенство и изобразите набор решений.

  1. [латекс] x + 2 <7 [/ латекс]
  2. [латекс] x + 8 \ le-7 [/ латекс]
  3. [латекс] x + 4> 13 [/ латекс]

Решения

  1. Чтобы решить неравенство x + 2 <7, вычтите 2 с обеих сторон неравенства. x + 2 — 2 <7 - 2
    Упростить: x <5
  2. Чтобы решить неравенство [латекс] x + 8 \ le – 7 [/ latex], вычтите 8 с обеих сторон неравенства. [латекс] x + 8 — 8 \ le-7 — 8 [/ latex] Упростить:
    [латекс] x \ ge – 15 [/ латекс]
  3. Чтобы решить неравенство x + 4> 13, вычтите 4 с обеих сторон неравенства. x + 4-4> 13-4 Упростить:
    x > 9

Рассмотрим проблему:
[латекс] \ frac {x} {5} \ le 3 [/ latex]

Чтобы найти решение, умножаем обе части на 5:
[latex] (5) \ frac {x} {5} \ le 3 (5) [/ latex]

Получаем
[латекс] х \ ле 15 [/ латекс].

Ответ неравенства может быть выражен четырьмя различными способами:

  1. Обозначение неравенства Ответ просто выражается как x <15.
  2. Обозначение набора Ответ: x | x <15. Вы читаете это как «набор всех значений x, таких, что x является действительным числом меньше 15».
  3. Интервальное обозначение использует скобки для обозначения диапазона значений в решении. Решение интервального обозначения для нашей задачи — (−∞, 15). Обозначение интервалов также использует понятие бесконечности ∞ и отрицательной бесконечности −∞.Круглые или открытые скобки «(» и «)» означают, что число рядом со скобкой не входит в набор решений. Квадратные или закрытые скобки «[» и «]» означают, что число рядом со скобкой включено в набор решений.
  4. График решения показывает решение в виде вещественной числовой прямой. Закрашенный кружок на номере указывает, что номер включен в набор решений. Белый кружок указывает на то, что номер не входит в набор. Для нашего примера здесь нарисован график решения.

Пример 6

  1. [−4,6] говорит, что решения — это все числа от −4 до 6 , включая −4 и 6.
  2. (8,24) говорит, что решением являются все числа от 8 до 24, но не включает числа 8 и 24.
  3. [3,12) говорит, что решением являются все числа от 3 до 12, включая 3, но не включая 12.
  4. (−5, ∞) говорит, что решением являются все числа больше −5, , исключая −5.
  5. (−∞, ∞) говорит, что решение — все действительные числа.

Решаем неравенство
[латекс] 2x \ ge 12 [/ latex] делением обеих сторон на 2:

[латекс] \ frac {2x} {2} \ ge \ frac {12} {2} [/ латекс]

Упростить:
[латекс] x \ ge 6 [/ латекс]

Давайте запишем решение в четырех различных обозначениях, которые вы только что выучили:

Обозначение неравенства [латекс] x \ ge 6 [/ латекс]
Обозначение набора [латекс] {x | x \ ge 6} [/ латекс]
Интервальное обозначение [латекс] [6, [/ латекс] ∞]
График решения

Мы решаем неравенство аналогично решению обычного уравнения.Мы можем складывать или вычитать числа по обе стороны неравенства. Мы также можем умножать или делить положительные числа по обе стороны неравенства, не меняя решения.

Что-то другое происходит, если мы умножаем или делим на отрицательные числа. В этом случае знак неравенства меняет направление.

Например, чтобы решить −3x <9

Делим обе части на –3. Знак неравенства меняется с <на>, потому что мы делим на отрицательное число.

[латекс] \ frac {-3x} {- 3} \ gt \ 6 \ frac {9} {- 3} [/ латекс]

[латекс] x \ gt \ -3 [/ латекс]

Мы можем объяснить, почему это происходит, на простом примере.Мы знаем, что два меньше трех, поэтому можем записать неравенство.

2 <3
Если мы умножим оба числа на −1, мы получим −2 и −3, но мы знаем, что −2 больше −3.

−2> −3

Вы видите, что умножение обеих сторон неравенства на отрицательное число привело к изменению направления знака неравенства. Это также происходит, если мы делим на отрицательное число.

Пример 7

Решите каждое неравенство. Приведите решение в обозначениях неравенства и интервалов.

  1. [латекс] 4x \ lt 24 [/ латекс] [латекс] \ displaystyle {4} {x} lt {24} [/ latex]
  2. [латекс] -9x \ le \ frac {3} {5} [/ латекс]
  3. [латекс] \ displaystyle- {5} {x} \ le {21} [/ latex]
  4. [латекс] \ displaystyle {12} {x} \ gt- {30} [/ latex]

Решения

  1. [latex] \ displaystyle {4} {x} \ lt {24} [/ latex] Разделите обе стороны на 4: [latex] \ displaystyle \ frac {{{4} {x}}} {{4}} \ lt \ frac {{24}} {{4}} [/ latex] Упростите, чтобы получить ответ:
    [latex] \ displaystyle {x} \ lt {6} [/ latex]
  2. [латекс] \ displaystyle- {9} {x} \ le- \ frac {{3}} {{5}} [/ latex] Разделите обе стороны на –9:
    [латекс] \ displaystyle \ frac {{- {9} {x}}} {{- {9}}} \ ge- \ frac {{3}} {{5}} (- \ frac {{1}} {{9}}) [/ latex] .Направление неравенства изменилось. Чтобы получить ответ, упростите:
    [latex] \ displaystyle {x} \ ge \ frac {{1}} {{15}} [/ latex]
  3. [латекс] \ displaystyle- {5} {x} \ le {21} [/ latex] Разделите обе стороны на –5:
    [латекс] \ displaystyle \ frac {{- {5} {x}}} {{ — {{5}}}} \ ge \ frac {{21}} {{- {{5}}}} [/ latex] Направление неравенства изменено. Упростите, чтобы получить ответ
    [latex] \ displaystyle {x} \ ge- \ frac {{21}} {{5}} [/ latex]
  4. [latex] \ displaystyle {12} {x} \ gt- {30} [/ latex] Разделите обе стороны на 12: [latex] \ displaystyle \ frac {{{12} {x}}} {{12}} \ gt- \ frac {{30}} {{12}} [/ latex] Упростите, чтобы получить ответ
    [latex] \ displaystyle {x} \ gt- \ frac {{5}} {{2}} [/ латекс]

Пример 8

Решите каждое неравенство.Приведите решение в обозначениях неравенств.

  1. [латекс] \ displaystyle \ frac {{x}} {{2}} \ gt {40} [/ latex]
  2. [латекс] \ displaystyle \ frac {{x}} {{- {3}}} \ le- {12} [/ latex]
  3. [латекс] \ displaystyle \ frac {{x}} {{25}} \ lt \ frac {{3}} {{2}} [/ latex]
  4. [латекс] \ displaystyle \ frac {{x}} {{- {7}}} \ ge {9} [/ latex]

Решения

  1. [латекс] \ displaystyle \ frac {{x}} {{2}} \ gt {40} [/ latex] Умножьте обе стороны на 2: [латекс] \ displaystyle {2} (\ frac {{x}} {{2}}) \ gt {40} ({2}) [/ latex] Упростить: [latex] \ displaystyle {x} \ gt {80} [/ latex]
  2. [латекс] \ displaystyle \ frac {{x}} {{- {3}}} \ le {12} [/ latex] Умножьте обе стороны на –3:
    [латекс] \ displaystyle- {3} (\ frac {{x}} {{- {3}}}) \ ge- {12} (- {3}) [/ latex] Направление неравенства изменено.Упростить: [latex] \ displaystyle {x} \ ge {36} [/ latex]
  3. .
  4. [латекс] \ displaystyle \ frac {{x}} {{25}} \ lt \ frac {{3}} {{2}} [/ latex] Умножьте обе стороны на 25 [латекс] \ displaystyle {25} ( \ frac {{x}} {{25}}) \ lt \ frac {{3}} {{2}} ({25}) [/ latex] Упростить:
    [латекс] \ displaystyle {x} \ lt \ гидроразрыв {{75}} {{2}} [/ latex] или 37,5
  5. [латекс] \ displaystyle \ frac {{x}} {{- {7}}} \ ge {9} [/ latex] Умножьте обе стороны на –7: [латекс] \ displaystyle (- {7}) \ frac {{x}} {{- {7}}} \ le {9} (- {7}) [/ latex] Направление неравенства изменено. Упростите: [latex] \ displaystyle {x} \ le- {63} [/ латекс]
  • Ответом на неравенство часто бывает интервал значений.Распространенные неравенства:
    • ge больше или равно
    • le меньше или равно
  • Решение неравенств с помощью сложения и вычитания работает так же, как решение уравнения. Чтобы решить, мы изолируем переменную на одной стороне уравнения.
  • Есть четыре способа изобразить неравенство:
    1. Обозначение уравнения x ge 2
    2. Установить обозначение x ge 2
    3. Интервальное обозначение [2, ∞) Закрытые скобки «[» и «]» означают включение, круглые скобки «(« и »)» означают исключение.
    4. График решения
  • При умножении или делении обеих частей неравенства на отрицательное число необходимо обратить неравенство.

ck12, Algebra, Linear Inequalities, «
Solving One-Step Inequalities», под лицензией CC BY-NC 3.0.

Решите линейные неравенства — элементарная алгебра

Цели обучения

К концу этого раздела вы сможете:

  • График неравенств на числовой прямой
  • Решите неравенства, используя свойства вычитания и сложения неравенства
  • Решите неравенства, используя свойства деления и умножения неравенства
  • Решить неравенства, требующие упрощения
  • Переведите в неравенство и решите

Прежде чем начать, пройдите тест на готовность.

  1. Перевести с алгебры на английский язык:.
    Если вы пропустили эту проблему, просмотрите (рисунок).
  2. Решение:
    Если вы пропустили эту проблему, просмотрите (рисунок).
  3. Решение:
    Если вы пропустили эту проблему, просмотрите (рисунок).
  4. Решение:
    Если вы пропустили эту проблему, просмотрите (рисунок).

Неравенства в графике на числовой прямой

Вы помните, что означает число, являющееся решением уравнения? Решение уравнения — это значение переменной, которое делает истинное утверждение при подстановке в уравнение.

А как насчет решения неравенства? Какое число сделало бы неравенство истинным? Вы думаете, « x может быть 4»? Это верно, но x тоже может быть 5, 20 или даже 3,001. Любое число больше 3 является решением неравенства.

Мы показываем решения неравенства на числовой прямой, закрашивая все числа справа от 3, чтобы показать, что все числа больше 3 являются решениями. Поскольку число 3 само по себе не является решением, мы помещаем 3 в открытую скобку.График представлен на (Рисунок). Обратите внимание, что используется следующее соглашение: голубые стрелки указывают в положительном направлении, а синие стрелки указывают в отрицательном направлении.

На этой числовой прямой изображено неравенство.

График неравенства очень похож на график, но теперь нам нужно показать, что 3 тоже является решением. Мы делаем это, помещая скобку в, как показано на (Рисунок).

На этой числовой прямой изображено неравенство.

Обратите внимание, что символ открытых скобок, (, показывает, что конечная точка неравенства не включена.Символ открытой скобки [, означает, что конечная точка включена.

График на числовой прямой:

ⓐⓑⓒ

График на числовой прямой: ⓐ ⓑ ⓒ

График на числовой прямой: ⓐ ⓑ ⓒ

Мы также можем представить неравенства, используя интервальную нотацию . Как мы видели выше, неравенство означает все числа больше 3. У решения этого неравенства нет верхней границы. В обозначении интервалов мы выражаем это как «бесконечность».Это не настоящее число. (Рисунок) показывает как числовую линию, так и обозначение интервалов.

Неравенство изображено на этой числовой строке и записано в интервальной записи.

Неравенство означает, что все числа меньше или равны 1. У этих чисел нет нижнего конца. Мы пишем в обозначениях интервалов как. Символ читается как «отрицательная бесконечность». (Рисунок) показывает как числовую линию, так и интервал.

Неравенство изображено на этой числовой строке и записано в интервальной записи.

Неравенства, числовые линии и обозначение интервалов

Вы заметили, как скобка или скобка в обозначении интервала соответствует символу на конце стрелки? Эти отношения показаны на (Рисунок).

В обозначении неравенств на числовой прямой и в обозначении интервалов используются аналогичные символы для обозначения конечных точек интервалов.

График в числовой строке и запись в интервальной записи.

ⓐⓑⓒ

График на числовой строке и запись в интервале записи:

ⓐⓑⓒ

График на числовой строке и запись в интервале записи:

ⓐⓑⓒ

Решите неравенства, используя свойства неравенства вычитания и сложения

Свойства равенства вычитания и сложения утверждают, что если две величины равны, когда мы прибавляем или вычитаем одну и ту же сумму из обеих величин, результаты будут одинаковыми.

Свойства равенства

Аналогичные свойства сохраняются и для неравенств.

Например, мы знаем, что −4 меньше 2.
Если мы вычтем 5 из обеих величин, будет ли левая часть
меньше правой?
Мы получаем −9 слева и −3 справа.
И мы знаем, что −9 меньше −3.
Знак неравенства остался прежним.

Аналогичным образом мы могли бы показать, что неравенство остается неизменным и для сложения.

Это приводит нас к свойствам неравенства на вычитание и сложение.

Свойства неравенства

Мы используем эти свойства для решения неравенств, выполняя те же действия, что и при решении уравнений. Решая неравенство, шаги будут выглядеть так:

Вычтите 5 с обеих сторон, чтобы изолировать.
Упростить.

Любое число больше 4 является решением этого неравенства.

Решите неравенство, нанесите решение на числовой прямой и запишите решение в интервальной записи.

Решите неравенство, нанесите решение на числовой прямой и запишите решение в интервальной записи.

Решите неравенство, нанесите решение на числовой прямой и запишите решение в интервальной записи.

Решение неравенств, используя свойства неравенства деления и умножения

Свойства равенства деления и умножения гласят, что если две величины равны, когда мы делим или умножаем обе величины на одинаковую величину, результаты также будут равны (при условии, что мы не делим на 0).

Свойства равенства

Существуют ли аналогичные свойства для неравенств? Что происходит с неравенством, когда мы делим или умножаем обе части на константу?

Рассмотрим числовые примеры.

Неравенство сохраняется, когда мы делим или умножаем на отрицательное число?

Когда мы делим или умножаем неравенство на положительное число, знак неравенства остается прежним. Когда мы делим или умножаем неравенство на отрицательное число, знак неравенства меняется на противоположный.

Вот свойства неравенства деления и умножения для удобства.

Свойства неравенства деления и умножения

Когда мы делим или умножаем неравенство на:

  • положительное число , неравенство остается таким же .
  • отрицательное число , неравенство отменяет .

Решите неравенство, нанесите решение на числовой прямой и запишите решение в интервальной записи.

Решите неравенство, нанесите решение на числовой прямой и запишите решение в интервальной записи.


Решите неравенство, нанесите решение на числовой прямой и запишите решение в интервальной записи.


Решите неравенство, нанесите решение на числовой прямой и запишите решение в интервальной записи.

Решите каждое неравенство, нанесите решение на числовой прямой и запишите решение в интервальной записи.


Решите каждое неравенство, нанесите решение на числовой прямой и запишите решение в интервальной записи.

Устранение неравенств

Иногда при решении неравенства переменная оказывается справа.Мы можем переписать неравенство в обратном порядке, чтобы переменная оказалась слева.

Думайте об этом так: «Если Ксавье выше Алекса, значит, Алекс ниже Ксавьера».

Решите неравенство, нанесите решение на числовой прямой и запишите решение в интервальной записи.

Решите неравенство, нанесите решение на числовой прямой и запишите решение в интервальной записи.

Решите неравенство, нанесите решение на числовой прямой и запишите решение в интервальной записи.

Решите неравенство, нанесите решение на числовой прямой и запишите решение в интервальной записи.

Решите неравенство, нанесите решение на числовой прямой и запишите решение в интервальной записи.

Решите неравенство, нанесите решение на числовой прямой и запишите решение в интервальной записи.

Устранение неравенств, требующих упрощения

Для устранения большинства неравенств потребуется более одного шага.Мы следуем тем же шагам, что и в общей стратегии решения линейных уравнений, но обязательно обращаем особое внимание во время умножения или деления.

Решите неравенство, нанесите решение на числовой прямой и запишите решение в интервальной записи.

Решите неравенство, нанесите решение на числовой прямой и запишите решение в интервальной записи.

Решите неравенство, нанесите решение на числовой прямой и запишите решение в интервальной записи.

Решите неравенство, нанесите решение на числовой прямой и запишите решение в интервальной записи.

Решите неравенство, нанесите решение на числовой прямой и запишите решение в интервальной записи.

Решите неравенство, нанесите решение на числовой прямой и запишите решение в интервальной записи.

Подобно тому, как некоторые уравнения являются тождествами, а некоторые — противоречиями, неравенства могут быть тождествами или противоречиями.Мы узнаем эти формы, когда у нас остаются только константы при решении неравенства. Если результатом является истинное утверждение, у нас есть личность. Если результатом является ложное утверждение, мы приходим к противоречию.

Решите неравенство, нанесите решение на числовой прямой и запишите решение в интервальной записи.

Решите неравенство, нанесите решение на числовой прямой и запишите решение в интервальной записи.

Решите неравенство, нанесите решение на числовой прямой и запишите решение в интервальной записи.

Решите неравенство, нанесите решение на числовой прямой и запишите решение в интервальной записи.

Решите неравенство, нанесите решение на числовой прямой и запишите решение в интервальной записи.

Решите неравенство, нанесите решение на числовой прямой и запишите решение в интервальной записи.

Переведите на неравенство и решите

Чтобы перевести английские предложения в выражения неравенства, нам нужно распознавать фразы, указывающие на неравенство.Некоторые слова просты, например «больше чем» и «меньше чем». Но другие не так очевидны.

Подумайте о фразе «по крайней мере» — что значит быть «не моложе 21 года»? Значит 21 или больше. Фраза «по крайней мере» означает «больше или равно».

(рисунок) показывает несколько распространенных фраз, указывающих на неравенство.

больше больше или равно меньше меньше или равно
больше — не менее меньше — не более
больше не менее имеет менее не более
превышает — это минимум ниже максимальное

Переведите и решите.Затем запишите решение в интервальной записи и нанесите график на числовой прямой.

Двенадцать раз c не более 96.

Переведите и решите. Затем запишите решение в интервальной записи и нанесите график на числовой прямой.

Двадцать раз y не более 100

Переведите и решите. Затем запишите решение в интервальной записи и нанесите график на числовой прямой.

Девять раз z не менее 135

Переведите и решите.Затем запишите решение в интервальной записи и нанесите график на числовой прямой.

Тридцать меньше x не меньше 45.

Переведите и решите. Затем запишите решение в интервальной записи и нанесите график на числовой прямой.

Девятнадцать менее p не менее 47

Переведите и решите. Затем запишите решение в интервальной записи и нанесите график на числовой прямой.

Четыре больше чем максимум 15.

Ключевые понятия

  • Свойство неравенства вычитания
    Для любых чисел a, b и c:
    , если тогда и
    , если тогда
  • Свойство сложения неравенства
    Для любых чисел a, b и c,
    , если тогда и
    , если тогда
  • Свойства неравенства деления и умножения y
    Для любых чисел a, b и c,
    if and, then and.
    если и, то и.
    если и, то и.
    если и, то и.
  • Когда мы делим или умножаем неравенство на a:
    • положительное число , неравенство остается таким же .
    • отрицательное число , неравенство отменяет .

Упражнения по разделам

Практика ведет к совершенству

Неравенства в графике на числовой прямой

В следующих упражнениях нарисуйте каждое неравенство на числовой прямой.

В следующих упражнениях нарисуйте каждое неравенство на числовой прямой и запишите в интервальной нотации.

Решите неравенства, используя свойства неравенства вычитания и сложения

В следующих упражнениях решите каждое неравенство, нанесите решение на числовой прямой и запишите решение в интервальной записи.

Решите неравенства, используя свойства неравенства деления и умножения

В следующих упражнениях решите каждое неравенство, нанесите решение на числовой прямой и запишите решение в интервальной записи.

Устранение неравенств, требующих упрощения

В следующих упражнениях решите каждое неравенство, нанесите решение на числовой прямой и запишите решение в интервальной записи.

Смешанная практика

В следующих упражнениях решите каждое неравенство, нанесите решение на числовой прямой и запишите решение в интервальной записи.

Перевести на неравенство и разрешить

В следующих упражнениях переведите и решите. Затем запишите решение в интервальной нотации и нанесите график на числовой прямой.

Четырнадцать раз d больше 56.

Девяносто раз c меньше 450.

В восемь раз z меньше, чем.

Десять раз y — самое большее.

Три более ч не менее 25.

Шесть больше k больше 25.

Десять меньше w не меньше 39.

Двенадцать меньше x не меньше 21.

Отрицательное пять раз r не более 95.

Дважды отрицательное значение s меньше 56.

Девятнадцать меньше b не больше.

Пятнадцать меньше, чем , а — минимум.

Повседневная математика

Безопасность Рост ребенка, h , должен быть не менее 57 дюймов, чтобы ребенок мог безопасно ездить на переднем сиденье автомобиля. Запишите это как неравенство.

Летчики-истребители Максимальная высота х летчика-истребителя составляет 77 дюймов. Запишите это как неравенство.

Лифты Общий вес пассажиров лифта, w , не может превышать 1 200 фунтов.Запишите это как неравенство.

Покупки Количество товаров, n , которые покупатель может иметь в полосе экспресс-оформления, не превышает 8. Запишите это как неравенство.

Письменные упражнения

Приведите пример из своей жизни, используя фразу «хотя бы».

Приведите пример из своей жизни, используя фразу «максимум».

Объясните, почему при решении необходимо обратить неравенство.

Объясните, почему при решении необходимо обратить неравенство.

Самопроверка

ⓐ После выполнения упражнений используйте этот контрольный список, чтобы оценить свое мастерство в достижении целей этого раздела.

ⓑ Что этот контрольный список говорит вам о вашем мастерстве в этом разделе? Какие шаги вы предпримете для улучшения?

Упражнения на повторение главы 2

Решение уравнений с использованием свойств равенства и сложения

Проверить решение уравнения

В следующих упражнениях определите, является ли каждое число решением уравнения.

Решите уравнения, используя свойства равенства равенства и вычитания и сложения

В следующих упражнениях решите каждое уравнение, используя свойство равенства вычитания.

В следующих упражнениях решите каждое уравнение, используя свойство сложения равенства.

В следующих упражнениях решите каждое уравнение.

Решите уравнения, требующие упрощения

В следующих упражнениях решите каждое уравнение.

Перевести в уравнение и решить

В следующих упражнениях переведите каждое английское предложение в алгебраическое уравнение, а затем решите его.

Сумма и 25.

Четыре меньше, чем 13.

Переводчик и решение приложений

В следующих упражнениях переведите в алгебраическое уравнение и решите.

Дочке Рошель 11 лет. Ее сын на 3 года младше. Сколько лет ее сыну?

Tan весит 146 фунтов. Минь весит на 15 фунтов больше, чем Тан. Сколько весит Мин?

Питер заплатил 9,75 фунтов стерлингов, чтобы пойти в кино, что составило 46 фунтов стерлингов.На 25 меньше, чем он заплатил, чтобы пойти на концерт. Сколько он заплатил за концерт?

Элисса заработала 152,84 фунта стерлингов на этой неделе, что на 21,65 евро больше, чем она заработала на прошлой неделе. Сколько она заработала на прошлой неделе?

Решите уравнения, используя свойства равенства деления и умножения

Решите уравнения, используя свойства равенства деления и умножения

В следующих упражнениях решите каждое уравнение, используя свойства равенства и умножения, и проверьте решение.

Решите уравнения, требующие упрощения

В следующих упражнениях решите каждое уравнение, требующее упрощения.

Перевести в уравнение и решить

В следующих упражнениях преобразуйте уравнение, а затем решите его.

143 является произведением и y .

Частное b и 9 равно.

Сумма q , и одна четвертая равна единице.

Разница s и одна двенадцатая составляет одну четвертую.

Переводчик и решение приложений

В следующих упражнениях переведите в уравнение и решите.

Рэй заплатил 21 евро за 12 билетов на ярмарке графства.Какова цена каждого билета?

Джанет платят 24 евро в час. Она слышала, что это то, чем платят Адаму. Сколько Адаму платят за час?

Решение уравнений с переменными и константами с обеих сторон

Решите уравнение с константами с обеих сторон

В следующих упражнениях решите следующие уравнения с константами с обеих сторон.

Решите уравнение с переменными с обеих сторон

В следующих упражнениях решите следующие уравнения с переменными с обеих сторон.

Решите уравнение с переменными и константами с обеих сторон

В следующих упражнениях решите следующие уравнения с переменными и константами с обеих сторон.

Использование общей стратегии решения линейных уравнений

Решение уравнений с использованием общей стратегии решения линейных уравнений

В следующих упражнениях решите каждое линейное уравнение.


Классифицируйте уравнения

В следующих упражнениях классифицируйте каждое уравнение как условное уравнение, тождество или противоречие, а затем сформулируйте решение.



противоречие; нет решения


удостоверение личности; все действительные числа

Решение уравнений с дробями и десятичными знаками

Решение уравнений с дробными коэффициентами

В следующих упражнениях решите каждое уравнение с дробными коэффициентами.

Решение уравнений с десятичными коэффициентами

В следующих упражнениях решите каждое уравнение с десятичными коэффициентами.

Решите линейные неравенства

Неравенства в графике на числовой прямой

В следующих упражнениях нарисуйте каждое неравенство на числовой прямой.

В следующих упражнениях нарисуйте каждое неравенство на числовой прямой и запишите в интервальной нотации.

Решите неравенства, используя свойства неравенства вычитания и сложения

В следующих упражнениях решите каждое неравенство, нанесите решение на числовой прямой и запишите решение в интервальной записи.

Решите неравенства, используя свойства неравенства деления и умножения

В следующих упражнениях решите каждое неравенство, нанесите решение на числовой прямой и запишите решение в интервальной записи.

Устранение неравенств, требующих упрощения

В следующих упражнениях решите каждое неравенство, нанесите решение на числовой прямой и запишите решение в интервальной записи.

Перевести на неравенство и разрешить

В следующих упражнениях переведите и решите.Затем запишите решение в интервальной записи и нанесите график на числовой прямой.

Пять больше z не больше 19.

Три меньше c не меньше 360.

Дважды отрицательное не более 8.

Повседневная математика

Опишите, как вы использовали две темы из этой главы в своей жизни за пределами урока математики в течение последнего месяца.

Практический тест, глава 2,

Определите, является ли каждое число решением уравнения.


5

В следующих упражнениях решите каждое уравнение.

противоречие; нет решения

Решите формулу для y
ⓐ, когда
ⓑ в целом

ⓐⓑ

В следующих упражнениях нарисуйте график на числовой прямой и запишите в интервальной записи.

В следующих упражнениях, решите каждое неравенство, нанесите решение на числовой прямой и запишите решение в интервальной записи.

В следующих упражнениях переведите уравнение или неравенство и решите.

4 меньше чем вдвое x равно 16.

Пятнадцать больше n не меньше 48.

Самуэль заплатил за бензин 25,82 фунта стерлингов на этой неделе, что составило 3 фунта стерлингов.На 47 меньше, чем он заплатил на прошлой неделе. Сколько он заплатил на прошлой неделе?

Дженна купила пальто на распродаже за 120 фунтов стерлингов, что было по первоначальной цене. Какова была первоначальная цена пальто?

; Первоначальная цена составляла 180 фунтов стерлингов.

Шон сел на автобус из Сиэтла в Бойсе, расстояние 506 миль. Если поездка длилась несколько часов, какова скорость автобуса?

.

Добавить комментарий

Ваш адрес email не будет опубликован. Обязательные поля помечены *